CDM revision Sem 2 Flashcards

1
Q

What is the classic triad for Behcet’s disease?

A

The classic triad in Behcet’s is oral ulcers, genital ulcers and uveitis. Venous thromboembolism is also seen.

How well did you know this?
1
Not at all
2
3
4
5
Perfectly
2
Q

What ia most severe manifestation of chronic Chagas’ disease?(how is transmitted?

A

Cardiomyopathy is the most frequent and most severe manifestation of chronic Chagas’ disease
Important for meLess important

Trypanosoma cruzi.

How well did you know this?
1
Not at all
2
3
4
5
Perfectly
3
Q

What is the most common cause of travellers diarrhea?

A

E-coli

How well did you know this?
1
Not at all
2
3
4
5
Perfectly
4
Q

Where does does pseudomonas cause infection and how is it treated?

A

Pseudomonas aeruginosa is a gram-negative rod that is an important cause of infections in immunocompromised or severely ill patients. It is a multi-drug resistant pathogen (in part aided by the ability to form biofilms) that frequently colonizes medical devices, and can secrete a wide host of virulence factors and exotoxins.

Pseudomonas commonly infects the airway, urinary tract and wounds or sites of entry for hospital devices/lines. Important diseases include the following:

Pneumonia: particularly ventilator-associated pneumonia, as well as pneumonia in Cystic fibrosis (80% are colonized, and once infection is established it can be very hard to treat)
Urinary tract infections: usually hospital acquired and associated with urinary tract catheterization or surgery.
Surgical wound and skin infections: skin lesions may include ecthyma gangrenosum
Sepsis in hospital/ nursing home inpatients
Infective endocarditis: especially in intravenous drug users or those with prosthetic heart valves
Ear infections: chronic otitis media as well as otitis externa (including malignant otitis externa)
Eye infections: bacterial keratitis and endophthalmitis (risk factors being trauma and wearing contact lenses)
Bones and joints: can cause osteomyelitis or septic arthritis, particularly in spine, pelvis and sternoclavicular joints. Being diabetic or an IVDU are risk factors.
Antibiotics which are effective against Pseudomonas:

Ciprofloxacin or levofloxacin (but not moxifloxacin) - note, this is the only oral anti-pseudomonal
Tazocin
Ceftazidime
Meropenem
Gentamicin
How well did you know this?
1
Not at all
2
3
4
5
Perfectly
5
Q

What investigations are done for typhoid?

A

Investigations: Note that the isolation of S.typhi is highest in the first week and becomes more difficult as time passes.

Blood culture - 80% positivity if two sets were taken.
bone marrow culture - most sensitive source.
Stool cultures
Widal test (for antibodies against Salmonella antigens), not very sensitive or specific.
Management:

Empirical antibiotics: Ciprofloxacin unless resistant. Alternatives include ceftriaxone, chloramphenicol or Azithromycin (if uncomplicated).
Supportive therapy: hydration, nutrition, antipyretics etc.
May need surgery for bowel perforation
Hygiene and close attention to hand washing must be emphasized to the patient and all close contacts.

How well did you know this?
1
Not at all
2
3
4
5
Perfectly
6
Q

What is the first line treatment of PCP?

A

Cotrimazole

How well did you know this?
1
Not at all
2
3
4
5
Perfectly
7
Q

How is Rheumatic fever diagnosed?

A

Acute or Chronic Rheumatic Fever - this is a multisystem autoimmune phenomenon affecting the heart, joints, skin, brain and subcutaneous tissue. It is caused by molecular mimicry, when shared epitopes between the host and group A strep lead to the immune system mistaking host tissues as ‘foreign,’- the most important being the Streptococcal M protein, which causes immune cross-reactivity with cardiac myosin. Patients commonly present with joint involvement (usually a migrating polyarthritis) as well as mitral valve regurgitation or stenosis. The modified Jones criteria is used for diagnosis (see below). It is rare in developed countries, but still commonly affects children (between 5-15 years) in developing countries - this is thought to be due to overcrowding, malnourishment and poor use of antibiotics to treat strep throat infections

Modified Jones criteria: 2 major or 1 major + 2 minor

Major criteria: Carditis, arthritis, erythema marginatum, Sydenham’s chorea, or subcutaneous nodules

Minor criteria: Fever, arthralgia, raised ESR/CRP, raised WCC, ECG showing heart block, previous rheumatic heart disease

How well did you know this?
1
Not at all
2
3
4
5
Perfectly
8
Q

Causes of pneumonia in homeless people: malnourished, alcohol or drug dependent, immunosuppressed

A

Homeless people: malnourished, alcohol or drug dependent, immunosuppressed
Mycobacterium tuberculosis
Aspiration pneumonia (infection with normal flora of mouth and anaerobes, also consider in any patient with an unsafe swallow or with depressed consciousness)
Klebsiella pneumoniae (causes ‘red-current jelly’ sputum, and commonly causes lung abscess formation and empyema)

How well did you know this?
1
Not at all
2
3
4
5
Perfectly
9
Q

How does Legionella pneumonia present?

A

Legionnaire’s disease is caused by the intracellular bacterium Legionella pneumophilia. It typically colonizes water tanks and hence questions may hint at air-conditioning systems or foreign holidays. Person-to-person transmission is not seen

Features
flu-like symptoms including fever (present in > 95% of patients)
dry cough
relative bradycardia
confusion
lymphopaenia
hyponatraemia
deranged liver function tests
pleural effusion: seen in around 30% of patients

Diagnosis
urinary antigen

Management
treat with erythromycin/clarithromycin

How well did you know this?
1
Not at all
2
3
4
5
Perfectly
10
Q

How is tapeworm infestation treated?

A

Praziquantel and Niclosamide

This is a description of Taeniasis or tapeworm. It can be contracted by eating uncooked pork or beef as the eggs are laid in meat. The patient describes seeing grapefruit seeds in his stool, these are proglottids which are tapeworm segments. It can be treated with anti-parasitic therapy.

How well did you know this?
1
Not at all
2
3
4
5
Perfectly
11
Q

How is rheumatic fever treated and what are the complications?

A

Management of rheumatic fever involves multiple goals:

Eradication of group-A beta-haemolytic streptococcal infection

STAT dose of IV Benzylpenicillin, with a ten day course of Phenoxymethylpenicillin to follow
Analgesia for arthritic symptoms

Aspirin or non-steroidal anti-inflammatory drugs (NSAIDs) such as ibuprofen or naproxen.
Aspirin should be used with caution in young children due to the small risk of Reye syndrome.
There is no evidence to suggest that NSAIDs help with outcomes related to carditis.
If carditis is complicated by heart failure

Glucocorticoids (e.g. Prednisolone) can provide benefit (NSAIDs should be stopped concurrently).
Diuretic treatment may also be necessary, and valve surgery if severe.
Sydenham’s chorea is self-limiting and does not require treatment, however Haloperidol or Diazepam may be used for distressing symptoms or risk of harm.

Erythema marginatum is associated with rheumatic fever is temporary and doesn’t require treatment, although antihistamines can help with pruritus.

Mechanism of valve complications

Rheumatic heart disease is a major cause of valvulopathy in children and young adults in the developing world. The most recent study in 2015 suggests that year there were over 30 million cases worldwide. Streptococcal antigens secondary to bacterial infection cross-reacts with the valve tissue, causing damage. Progressive damage commonly occurs in the years following acute rheumatic fever.

Mitral disease occurs in 70% of cases and is the most common affected valve; aortic valves are involved in 40% (most commonly regurgitation), tricuspid valves in 10% and pulmonary valves in 2%.

How well did you know this?
1
Not at all
2
3
4
5
Perfectly
12
Q

most commonly diagnosed sexually transmitted infection in the UK?

A

Chlamydia

How well did you know this?
1
Not at all
2
3
4
5
Perfectly
13
Q

How is toxoplasmosis treated?

A

Immunocompromised patients with toxoplasmosis are treated with pyrimethamine plus sulphadiazine

How well did you know this?
1
Not at all
2
3
4
5
Perfectly
14
Q

Who develops CMV retinitis?

A

CMV retinitis
common in HIV patients with a low CD4 count (< 50)
presents with visual impairment e.g. ‘blurred vision’. Fundoscopy shows retinal haemorrhages and necrosis, often called ‘pizza’ retina
IV ganciclovir is the treatment of choice

Cytomegalovirus (CMV) infection is important to consider in renal transplant patients.

How well did you know this?
1
Not at all
2
3
4
5
Perfectly
15
Q

First line treatment for gonorrhoea and chlamydia?

A

Gonorrhoea Intramuscular ceftriaxone

Chlamydia Doxycycline or azithromycin

How well did you know this?
1
Not at all
2
3
4
5
Perfectly
16
Q

What are the features and complications of mycoplasma infection?

A

Mycoplasma pneumoniae is a cause of atypical pneumonia which often affects younger patients. It is associated with a number of characteristic complications such as erythema multiforme and cold autoimmune haemolytic anaemia. Epidemics of Mycoplasma pneumoniae classically occur every 4 years. It is important to recognise atypical pneumonia as it may not respond to penicillins or cephalosporins due to it lacking a peptidoglycan cell wall.

Features
the disease typically has a prolonged and gradual onset
flu-like symptoms classically precede a dry cough
bilateral consolidation on x-ray
complications may occur as below

Complications
cold agglutins (IgM): may cause an haemolytic anaemia, thrombocytopenia
erythema multiforme, erythema nodosum
meningoencephalitis, Guillain-Barre syndrome and other immune-mediated neurological diseases
bullous myringitis: painful vesicles on the tympanic membrane
pericarditis/myocarditis
gastrointestinal: hepatitis, pancreatitis
renal: acute glomerulonephritis

Investigations
diagnosis is generally by Mycoplasma serology
positive cold agglutination test

Management
doxycycline or a macrolide (e.g. erythromycin/clarithromycin)

How well did you know this?
1
Not at all
2
3
4
5
Perfectly
17
Q

How is hospital acquired pneumonia treated?

A

Within 5 days of admission: co-amoxiclav or cefuroxime
More than 5 days after admission: piperacillin with tazobactam OR a broad-spectrum cephalosporin (e.g. ceftazidime) OR a quinolone (e.g. ciprofloxacin)

How well did you know this?
1
Not at all
2
3
4
5
Perfectly
18
Q

Name 4 radiological (x-ray) signs of rheumatoid arthritis of the hands?

A
Narrowing of joint space
Marginal erosions
Periarticular osteopenia / osteoporosis
Irregular joint surface
Deformity / subluxation / dislocation / malalignment
Soft tissue swelling
How well did you know this?
1
Not at all
2
3
4
5
Perfectly
19
Q

Name 4 organs and the associated pathology (not including any in the musculoskeletal system) which manifest as part of the systemic disease process of rheumatoid arthritis?

A

Skin (Rheumatoid nodules)
Eyes (episcleritis / kerato-conjunctivitis sicca)
Heart (pericarditis)
Lungs (rheumatoid nodules in lungs)
Spleen (Splenomegaly, Felty’s syndrome)
Blood (normochromic normocytic anaemia ; iron-deficiency anaemia; anaemia of chronic disease)
Renal (renal impairment / amyloidosis)
Gastrointestinal (stomach ulcers, GI bleeding, oesophagitis, amyloidosis)

How well did you know this?
1
Not at all
2
3
4
5
Perfectly
20
Q

What is Pityriasis rosea and how does it present?

A

Pityriasis rosea is a common rash which often occurs after an upper respiratory tract infection and is thought to have a viral cause (HHV 6/7).

It is characterized by a preceding herald patch - a single, large, discoid (coin-shaped), erythematous patch. This patch classically has a ‘collarette’ of scale just inside the edge of the lesion. A few days later a widespread rash appears across the trunk consisting of multiple small, erythematous, scaly patches (similar but smaller than the herald patch). These lesions are classically distributed across the trunk in a ‘christmas tree’ pattern.

Pityriasis rosea is self-limiting and benign. No treatment is required and it will usually resolve over a few weeks.

May need

Topical steroids and emollients. UV therapy is sometimes used.

How well did you know this?
1
Not at all
2
3
4
5
Perfectly
21
Q

Which cells are squamous cell carcinomas derived from and what are the commonest sites?What is the biggest risk factor

A

Keratinocytes
Backs of hands, face, edge of scars

Sun damage or sun exposure

How well did you know this?
1
Not at all
2
3
4
5
Perfectly
22
Q

What Investigations are done for Sjorgens

A

Blood tests:

Markers of inflammation are usually raised.
Autoantibodies:
Most patients are ANA positive but this is not specific for Sjogren’s syndrome.
Anti-Ro and Anti-La Autoantibodies are both specific for Sjogren’s syndrome.
Special tests for Sjogren’s syndrome

Schirmer’s test - this demonstrates reduced tear production using a strip of filter paper on the lower eyelid: wetting of <5mm is positive.
Rose bengal staining - this demonstrates keratitis due to conjunctivitis sicca when using a slit-lamp.
Salivary flow rate monitoring - this demonstrates xerostomia using a radiolabelled dye (uptake and excretion by the salivary gland is slowed).
Salivary gland biopsy - histology may confirm the diagnosis.

How well did you know this?
1
Not at all
2
3
4
5
Perfectly
23
Q

How can polymyalgia be treated and what Investigations need to be done before hand?

A

The treatment for polymyalgia rheumatica is steroids.

Unlike giant cell arteritis, steroids do not need to be started straight away giving time for an initial work-up to minimise risks of steroid use.

After initial investigations, before commencing treatment for Polymyalgia rheumatica the following work up should be done:

Prevention and treatment of steroid induced osteoporosis: DEXA scan those with high fracture risk. DEXA scanning is recommended in those without high fracture risk too.
Screening for increased risk of adverse reaction to steroids before starting: diabetes mellitus, hypertension, history of peptic ulcer, osteoporosis, mental health history.

How well did you know this?
1
Not at all
2
3
4
5
Perfectly
24
Q

What are the extra articular manifestation of osteoarthritis?

A

Extra-articular features:

Rheumatoid arthritis (RA) is a multisystem inflammatory disease and can affect most organs.

General:
Patients often feel generally unwell with RA, including low grade fevers, weight loss, and fatigue.

Haematological:

Anaemia of chronic disease - this is common in RA (note that most DMARDs can also cause cytopenias as a side effect).
Splenomegaly including Felty’s syndrome (triad of RA, splenomegaly and neutropenia).
Amyloidosis – most organs can be affected, classically the kidneys causing nephrotic syndrome.
Generalised lymphadenopathy.
Dermatological:

Rheumatoid nodules (firm, dark skin nodules, usually around sites of inflammation).
Small vessel vasculitis causing nailbed infarcts and arterial leg ulcers.
Raynaud’s syndrome.
Ophthalmic:

Keratoconjunctivitis sicca (dry eyes) – this can occur on its own, or as part of secondary Sjogren’s syndrome with oral, genital and gastric ulcers.
Episcleritis and scleritis.
Respiratory

Pleural effusions containing rheumatoid factor.
Rheumatoid nodules may be seen on chest x-ray, and are asymptomatic.
Pneumonitis leading to pulmonary fibrosis (note that this can also be a side effect of methotrexate).
Orthopaedic:

Osteoporosis.
Cardiac:

Pericardial effusions containing rheumatoid factor.
Strong risk factor for cardiovascular disease.
Neurological:

Peripheral neuropathy.

How well did you know this?
1
Not at all
2
3
4
5
Perfectly
25
Q

What are the risk factors for development of basal cell carcinoma?

A

Immunosuppression
UV exposure
Family history of skin ca

How well did you know this?
1
Not at all
2
3
4
5
Perfectly
26
Q

What are the risk factors for pseudo gout?

A

A diagnosis of pseudogout can be tricky to make. Pseudogout typically affects elderly women, and the joints that are normally affected are the wrists and knees.

The associations are hyperparathyroidism, hypothyroidism, hypophosphataemia, haemochromatosis and Wilson’s disease.

On polarised light microscopy, psedogout presents with positively birefringent rhomboid crystals.

How well did you know this?
1
Not at all
2
3
4
5
Perfectly
27
Q

What NSAIDS are safer to use if there is a risk of GI ulceration?

A

COX-2 inhibitors, also known as coxibs, such as celocoxib are a newer alternative to traditional NSAIDs like ibuprofen or naproxen. Coxibs selectively inhibit COX-2 which confers a lower risk of GI ulceration and so may be preferred in patients at high risk of GI ulceration.

However, shortly after they were first released, the coxib rofecoxib was discontinued because it was found to increased the risk of cardiovascular events to a significantly higher extent than regular NSAIDs. This lead to a lack of uptake in coxib use, but more recent research has found that other coxibs, particularly moderate dose celocoxib, confers an equivalent cardiovascular risk to normal NSAIDs and are therefore safe to use in place of regular NSAIDs where necessary.

How well did you know this?
1
Not at all
2
3
4
5
Perfectly
28
Q

What complication to the eyes can occur due to hydroxychloroquine use?

A

bullseye maculopathy-reduced visual acuity and colour vision
annular lesions bilaterally around macular on fundoscope

if suspected stop drug

How well did you know this?
1
Not at all
2
3
4
5
Perfectly
29
Q

How is Turner’s syndrome inherited?

A

This is a typical presentation of Turner’s syndrome. She has a webbed neck and
short stature which represent characteristic physical appearance. Infertility
is usually how these patients present. Genotype is XO and majority of foetuses
with this genetic make-up will spontaneously abort. As such, a live Turner’s
syndrome lady is most likely to be caused by mosaicism. Some cells are XO,
some are normal (XX).

How well did you know this?
1
Not at all
2
3
4
5
Perfectly
30
Q

The physiological process of X inactivation in an XX

female leads to creation of which cellular structure?

A

The normal state of genetic material in cells compromises two active sets of
autosomes (the non-sex chromosomes) and one active X chromosome in the female,
or in males who have multiple X chromosomes (Klinefelter’s syndrome). The X
chromosome that is “inactivated” becomes condensed, meaning it cannot be
replicated (hence inactivated) and this creates the Barr Body - the condensed
chromosome pushed against the nuclear membrane. They are rendered inactive in
a process called lyonisation.

How well did you know this?
1
Not at all
2
3
4
5
Perfectly
31
Q

What diagnostic tests can be used to identify if a foetus has Down syndrome?

A

As she has booked in at 9 weeks, you can carry
out chorionic villous sampling, which is undertaken at 10-14 weeks. It
involves taking a small sample of cells called chorionic villi from theplacenta where it attaches to the wall of the uterus. Genetic analysis can
then be carried out on these cells to determine the karyotype.

If she had presented later, amniocentesis can be undertaken at 13-15 weeks;
here, a sample of amniotic fluid, not placenta, is taken and tested.
Ultrasound to determine the ‘nuchal thickness’ and blood tests such as beta
hCG and PAPP-A are routinely used to screen for Down’s syndrome, but do not
involve obtaining a placental sample and are also screening tests, rather than
being diagnostic.

How well did you know this?
1
Not at all
2
3
4
5
Perfectly
32
Q

How is Osteogenesis Imperfects inherited?

A

Osteogenesis imperfecta is an autosomal dominant condition of collagen,
resulting in bone material being more supple and so fractures occur more
commonly. The type 1 procollagen compromises two chains encoded by COL1A1 gene
and one chain encoded for by COL1A2. A mutation in either of these genes
results in disruption of the whole fibril.

It’s mechanism of dominant inheritance is said to be the dominant negativeeffect. This is where the product of a defective gene (e.g. in this case,
either COL1A1 or COL1A2) interferes with the action of the normal allele (e.g.
here, whichever of COL1A1 or COL1A2 is not mutated). These usually need to
form units called “multimers” - proteins that work together to form a complex
structure.

How well did you know this?
1
Not at all
2
3
4
5
Perfectly
33
Q

What TB drug is most likely to cause hepatotoxicity?

A

Isoniazid more likely to cause hepatotoxicity than rifampicin

How well did you know this?
1
Not at all
2
3
4
5
Perfectly
34
Q

When should BCG vaccine be offered

A

Give BCG vaccination

According to NICE guidelines, BCG vaccination should be given to tuberculin skin test negative (mantoux negative) contacts of patients with confirmed pulmonary and laryngeal TB, who have not been previously vaccinated and are under the age of 35 or are over the age of 35 and work in healthcare.

This patient had a negative test with only 1mm induration, a positive result would have been 5mm or more.

How well did you know this?
1
Not at all
2
3
4
5
Perfectly
35
Q

What are the adverse effects of Ciprofloxacin

A

Ciprofloxacin

This is associated with increased risk of Achilles tendinopathy and tendon rupture, although this is rare. It may also cause lowering of seizure threshold, prolongation of QT interval and Clostridium difficile colitis.

How well did you know this?
1
Not at all
2
3
4
5
Perfectly
36
Q

What is the most common cause of food poisoning in the UK?

A

Campylobacter jejuni

This is the most common cause of food poisoning in the UK, and is usually associated with eating contaminated or undercooked chicken. Summer BBQs are classically associated small outbreaks or groups of cases. Diarrhoea is often dysenteric (containing blood and mucus) due to bacterial invasion of the intestinal mucosa

How well did you know this?
1
Not at all
2
3
4
5
Perfectly
37
Q

What is the first line empirical therapy for Neutropenic sepsis?

A

DRABCDE approach

If low risk can give oral antibiotics (quinolone + co-amoxiclav)

Features suggesting low risk:
Hemodynamically stable
Doesn't have acute leukemia
No organ failure
No soft tissue infection
No indwelling lines

For most patients, they need empirical IV treatment with piperacillin and tazobactam (tazocin), with added coverage for MRSA or gram-negatives if thought at risk. A macrolide should also be added if diagnosed with pneumonia (to cover atypical organisms)

May need 2-hourly assessments

Daily measures of fever and baseline bloods until the patient is apyrexial and neutrophil count above 0.5x10^9

When the neutrophil count is normal, has been afebrile for 48 hours and blood tests have normalized, antibiotics can be stopped.

Prophylaxis with a fluoroquinolone can be offered

How well did you know this?
1
Not at all
2
3
4
5
Perfectly
38
Q

What Investigations are done for TB?

A

Investigations

Chest X-ray
Sputum samples for culture and sensitivity testing (at least three needed - may need to consider lavage or sputum induction if cannot produce)
Samples from non-pulmonary sites: may need biopsy and needle aspiration
Samples are stained with Ziehl-Neelsen or Auramine staining for direct microscopy
For culture, Lowenstein-Jensen (LJ) media is needed.
PCR - GeneXpert, rapid results with additional drug sensitivity tested as well.
Interferon-Gamma Release Assays (IGRAs) - cannot tell difference between latent or active TB, and not useful in very young or immunosuppressed patients. Results are obtained rapidly, however, and are not affected by prior BCG.
Mantoux test - usually offered to contacts of infected patients. Positive in those who have had the BCG vaccine.
Management

How well did you know this?
1
Not at all
2
3
4
5
Perfectly
39
Q

What is Psoa’s sign”?

A

Severe pain whenever the left hip is hyperextended.

How well did you know this?
1
Not at all
2
3
4
5
Perfectly
40
Q

How can coagulase negative staph IE be treated?

A

Vancomycin
32%
Coagulase-negative staphylococci are the most common cause of endocarditis associated with hospital in-dwelling lines. Glycopeptides are first-line treatment

How well did you know this?
1
Not at all
2
3
4
5
Perfectly
41
Q

What are examples of live vaccines

A
Live attenuated vaccines
BCG
MMR
oral polio
yellow fever
oral typhoid
How well did you know this?
1
Not at all
2
3
4
5
Perfectly
42
Q

How can hospital acquired pneumonia be treated?

A

Within 5 days of admission: co-amoxiclav or cefuroxime
More than 5 days after admission: piperacillin with tazobactam OR a broad-spectrum cephalosporin (e.g. ceftazidime) OR a quinolone (e.g. ciprofloxacin)

How well did you know this?
1
Not at all
2
3
4
5
Perfectly
43
Q

What is th risk of infection following splenectomy and how can this be managed?

A

Organisms causing post splenectomy sepsis:
Streptococcus pneumoniae
Haemophilus influenzae
Meningococci
Important for meLess important
Encapsulated organisms carry the greatest pathogenic risk following splenectomy. The effects of sepsis following splenectomy are variable. This may be the result of small isolated fragments of splenic tissue that retain some function following splenectomy. These may implant spontaneously following splenic rupture (in trauma) or be surgically implanted at the time of splenectomy.

Management

Key recommendations
All those with hyposplenism or may become so (such as prior to an elective splenectomy) should receive pneumococcal, Haemophilus type b and meningococcal type C vaccines. These should be administered 2 weeks prior to splenectomy or two weeks following splenectomy. The vaccine schedule for meningococcal disease essentially consists of a dose of Men C and Hib at 2 weeks and then a dose of the MenACWY vaccine one month later. Those aged under 2 may require a booster at 2 years. A dose of pneumococcal polyvalent polysaccharide vaccine (PPV) is given at two weeks. A conjugated vaccine (PCV) is offered to young children. The PCV is more immunogenic but covers fewer serotypes. Boosting PPV is either guided by serological measurements (where available) or by routine boosting doses at 5 yearly intervals.
Annual influenza vaccination is recommended in all cases
Antibiotic prophylaxis is offered to all. The risk of post-splenectomy sepsis is greatest immediately following splenectomy and in those aged less than 16 years or greater than 50 years. Individuals with a poor response to pneumococcal vaccination are another high-risk group. High-risk individuals should be counselled to take penicillin or macrolide prophylaxis. Those at low risk may choose to discontinue therapy. All patients should be advised about taking antibiotics early in the case of intercurrent infections.
Asplenic individuals travelling to malaria endemic areas are at high risk and should have both pharmacological and mechanical protection.

Dosing

Penicillin V 500mg BD or amoxicillin 250mg BD

How well did you know this?
1
Not at all
2
3
4
5
Perfectly
44
Q

When should tetanus vaccine be given?

A

Patients with an uncertain tetanus vaccination history should be given a booster vaccine + immunoglobulin, unless the wound is very minor and < 6 hours old
Important for meLess important
In this case, the most appropriate course of action is to give a tetanus booster and tetanus immunoglobulin. This man has an unknown vaccination, a tetanus prone wound (occurred whilst gardening) and it is more than 6 hours old.

Metronidazole antibiotic is the treatment for tetanus rather than being used as a prophylactic agent.

A tetanus booster alone is sufficient if a patient has a tetanus prone wound with a full vaccination history but the last dose being more than 10 years ago.

Tetanus immunoglobulin can be used to treat proven cases of tetanus. In this case, as the vaccination history is not known a tetanus booster and immunoglobulin should be given.

Simple wound cleaning can be used if a patient has had a full course of tetanus vaccines, with the last dose less than 10 years ago, irrespective of the severity of the wound. The vaccination history is not known in this case so wound cleaning only would not be appropriate

How well did you know this?
1
Not at all
2
3
4
5
Perfectly
45
Q

What is the first line treatment for cellulitis

A

The BNF recommends flucloxacillin as first-line treatment for mild/moderate cellulitis. Clarithromycin, erythromycin (in pregnancy) or doxycyline is recommended in patients allergic to penicillin.

NICE recommend that patients severe cellulitis should be offered co-amoxiclav, cefuroxime, clindamycin or ceftriaxone.

How well did you know this?
1
Not at all
2
3
4
5
Perfectly
46
Q

What adverse reaction is seen with the use of metronidazole?

A

The combination of metronidazole and ethanol can cause a disulfiram-like reaction. Clinical features of this include head and neck flushing, nausea and vomiting, sweatiness, headache and palpitations.

Cefoperazone, a cephalosporin, is also associated with a disulfiram-like reaction to alcohol.

How well did you know this?
1
Not at all
2
3
4
5
Perfectly
47
Q

How can Parvovirus B19 trigger an aplastic crisis ?

A

Parvovirus B19 is the most likely infective cause of an aplastic anaemia crisis in patients with sickle cell disease
Important for meLess important
Erythema infectiosum is caused by parvovirus B19, a virus that reduces erythropoiesis for about a week. Those who already have a chronic haemolytic anaemia (e.g. sickle cell disease) are at high risk of aplastic anaemia.

How well did you know this?
1
Not at all
2
3
4
5
Perfectly
48
Q

What is the first line treatment for syphilis and what reaction is seen?

A

Management
intramuscular benzathine penicillin is the first-line management
alternatives: doxycycline
the Jarisch-Herxheimer reaction is sometimes seen following treatment
fever, rash, tachycardia after the first dose of antibiotic
in contrast to anaphylaxis, there is no wheeze or hypotension
it is thought to be due to the release of endotoxins following bacterial death and typically occurs within a few hours of treatment
No treatment is needed other than antipyretics if required

How well did you know this?
1
Not at all
2
3
4
5
Perfectly
49
Q

What organisms can cause necrotising fasciitis

A

Necrotising fasciitis is a medical emergency that is difficult to recognise in the early stages.

It can be classified according to the causative organism:
type 1 is caused by mixed anaerobes and aerobes (often occurs post-surgery in diabetics). This is the most common type
type 2 is caused by Streptococcus pyogenes

Risk factors
skin factors: recent trauma, burns or soft tissue infections
diabetes mellitus
the most common preexisting medical condition
particularly if the patient is treated with SGLT-2 inhibitors
intravenous drug use
immunosuppression

The most commonly affected site is the perineum (Fournier’s gangrene).

Features
acute onset
pain, swelling, erythema at the affected site
often presents as rapidly worsening cellulitis with pain out of keeping with physical features
extremely tender over infected tissue with hypoaesthesia to light touch
skin necrosis and crepitus/gas gangrene are late signs
fever and tachycardia may be absent or occur late in the presentation

Management
urgent surgical referral debridement
intravenous antibiotics

Prognosis
average mortality of 20%

How well did you know this?
1
Not at all
2
3
4
5
Perfectly
50
Q

What post exposure prophylaxis is given in Hep A,B,C and HIV

A

Hepatitis A
Human Normal Immunoglobulin (HNIG) or hepatitis A vaccine may be used depending on the clinical situation

Hepatitis B
HBsAg positive source: if the person exposed is a known responder to HBV vaccine then a booster dose should be given. If they are in the process of being vaccinated or are a non-responder they need to have hepatitis B immune globulin (HBIG) and the vaccine
unknown source: for known responders the green book advises considering a booster dose of HBV vaccine. For known non-responders HBIG + vaccine should be given whilst those in the process of being vaccinated should have an accelerated course of HBV vaccine

Hepatitis C
monthly PCR - if seroconversion then interferon +/- ribavirin

HIV
the risk of HIV transmission depends heavily on the incident (e.g. needle stick, type of sexual intercourse, human bite etc) and the current viral load of the patient
please see the BHIVA link for charts which outline the risk depending on the incident. Generally, low-risk incidents such as human bites don’t require post-exposure prophylaxis
a combination of oral antiretrovirals (e.g. Tenofovir, emtricitabine, lopinavir and ritonavir) as soon as possible (i.e. Within 1-2 hours, but may be started up to 72 hours following exposure) for 4 weeks
serological testing at 12 weeks following completion of post-exposure prophylaxis
reduces risk of transmission by 80%

Varicella zoster
VZIG for IgG negative pregnant women/immunosuppressed

How well did you know this?
1
Not at all
2
3
4
5
Perfectly
51
Q

How does CSF of TB meningitis look?

A

The table below summarises the characteristic cerebrospinal fluid (CSF) findings in meningitis:

Bacterial Viral Tuberculous Fungal
Appearance Cloudy Clear/cloudy Slight cloudy, fibrin web Cloudy
Glucose Low (< 1/2 plasma) 60-80% of plasma glucose* Low (< 1/2 plasma) Low
Protein High (> 1 g/l) Normal/raised High (> 1 g/l) High
White cells 10 - 5,000 polymorphs/mm³ 15 - 1,000 lymphocytes/mm³ 30 - 300 lymphocytes/mm³ 20 - 200 lymphocytes/mm³

The Ziehl-Neelsen stain is only 20% sensitive in the detection of tuberculous meningitis and therefore PCR is sometimes used (sensitivity = 75%)

*mumps is unusual in being associated with a low glucose level in a proportion of cases. A low glucose may also be seen in herpes encephalitis

How well did you know this?
1
Not at all
2
3
4
5
Perfectly
52
Q

What are the features of congenital syphilis?

A

Features of congenital syphilis
blunted upper incisor teeth (Hutchinson’s teeth), ‘mulberry’ molars
rhagades (linear scars at the angle of the mouth)
keratitis
saber shins
saddle nose
deafness

How well did you know this?
1
Not at all
2
3
4
5
Perfectly
53
Q

How is chlamydia diagnosed and treated?

A

Chlamydia is the most commonly diagnosed sexually transmitted infection. It can be asymptomatic and if left untreated can cause complications such as pelvic inflammatory disease and infertility. It is diagnosed using nucleic acid amplification tests (NAATs). Women can do a self swab and men can do a first pass urine sample. Gonorrhoea is also diagnosed using NAATs.

Treatment of Chlamydia is with azithromycin or doxycycline.

Chlamydia is the most prevalent sexually transmitted infection in the UK and is caused by Chlamydia trachomatis, an obligate intracellular pathogen. Approximately 1 in 10 young women in the UK have Chlamydia. The incubation period is around 7-21 days, although it should be remembered a large percentage of cases are asymptomatic

Features
asymptomatic in around 70% of women and 50% of men
women: cervicitis (discharge, bleeding), dysuria
men: urethral discharge, dysuria

Potential complications
epididymitis
pelvic inflammatory disease
endometritis
increased incidence of ectopic pregnancies
infertility
reactive arthritis
perihepatitis (Fitz-Hugh-Curtis syndrome)

Investigation
traditional cell culture is no longer widely used
nuclear acid amplification tests (NAATs) are now the investigation of choice
urine (first void urine sample), vulvovaginal swab or cervical swab may be tested using the NAAT technique
for women: the vulvovaginal swab is first-line
for men: the urine test is first-line
Chlamydiatesting should be carried out two weeks after a possible exposure

Screening
in England the National Chlamydia Screening Programme is open to all men and women aged 15-24 years
the 2009 SIGN guidelines support this approach, suggesting screening all sexually active patients aged 15-24 years
relies heavily on opportunistic testing
Management
doxycycline (7 day course) if first-line
this is now preferred to azithromycin due to concerns about Mycoplasma genitalium. This infection is often coexistant in patients with Chlamydia and there is evidence of rising levels of macrolide resistance, hence why doxycycline is preferred
if doxycycline is contraindicated / not tolerated then either azithromycin (1g od for one day, then 500mg od for two days) should be used
if pregnant then azithromycin, erythromycin or amoxicillin may be used. The SIGN guidelines suggest azithromycin 1g stat is the drug of choice ‘following discussion of the balance of benefits and risks with the patient’
patients diagnosed with Chlamydia should be offered a choice of provider for initial partner notification - either trained practice nurses with support from GUM, or referral to GUM
for men with urethral symptoms: all contacts since, and in the four weeks prior to, the onset of symptoms
for women and asymptomatic men all partners from the last six months or the most recent sexual partner should be contacted
contacts of confirmed Chlamydia cases should be offered treatment prior to the results of their investigations being known (treat then test)

How well did you know this?
1
Not at all
2
3
4
5
Perfectly
54
Q

How can different causes of meningitis be differentiated by microbiology

A

The clinical presentation raises a suspicion of meningitis. Streptococcus pneumoniae, Neisseria meningitidis, Mycobacterium tuberculosis and Cryptococcus neoformans are causative organisms of meningitis. The India ink stain on cerebrospinal fluid (CSF) analysis points towards Cryptococcus neoformans as the causative organism. Streptococcus pneumoniae would have a gram-positive stain while Neisseria meningitidis would have a gram-negative stain. Mycobacterium tuberculosis would have a Ziehl–Neelsen (acid-fast) stain. In toxoplasmosis, the head CT usually shows single or multiple ring enhancing lesions, and mass effect may be seen

How well did you know this?
1
Not at all
2
3
4
5
Perfectly
55
Q

What is red mans syndrome

A

Red man syndrome is associated with rapid intravenous infusion vancomycin. It is a common adverse reaction of intravenous vancomycin use and is a distinct entity from anaphylaxis due to vancomycin use. Typical symptoms include redness, pruritus and a burning sensation, predominantly in the upper body (face, neck and upper chest). Severe cases can be associated with hypotension and chest pain.

The pathophysiology of red man syndrome is attributed to vancomycin-related activation of mast cells with release of histamine.

The management of red man syndrome involves cessation of the infusion, and when symptoms have resolved, recommencement at a slower rate. In patients who are more symptomatic antihistamines can be administered, and may require intravenous fluids if the syndrome is associated with hypotension.

How well did you know this?
1
Not at all
2
3
4
5
Perfectly
56
Q

How can MRSA be treated and cleared?

A

Methicillin-resistant Staphylococcus aureus (MRSA) was one of the first organisms which highlighted the dangers of hospital-acquired infections.

Who should be screened for MRSA?
all patients awaiting elective admissions (exceptions include day patients having terminations of pregnancy and ophthalmic surgery. Patients admitted to mental health trusts are also excluded)
from 2011 all emergency admissions will be screened

How should a patient be screened for MRSA?
nasal swab and skin lesions or wounds
the swab should be wiped around the inside rim of a patient’s nose for 5 seconds
the microbiology form must be labelled ‘MRSA screen’

Suppression of MRSA from a carrier once identified

nose: mupirocin 2% in white soft paraffin, tds for 5 days
skin: chlorhexidine gluconate, od for 5 days. Apply all over but particularly to the axilla, groin and perineum

The following antibiotics are commonly used in the treatment of MRSA infections:
vancomycin
teicoplanin
linezolid

Some strains may be sensitive to the antibiotics listed below but they should not generally be used alone because resistance may develop:
rifampicin
macrolides
tetracyclines
aminoglycosides
clindamycin

Relatively new antibiotics such as linezolid, quinupristin/dalfopristin combinations and tigecycline have activity against MRSA but should be reserved for resistant cases

How well did you know this?
1
Not at all
2
3
4
5
Perfectly
57
Q

Antibiotic guidelines

A

Condition Recommended treatment
Exacerbations of chronic bronchitis Amoxicillin or tetracycline or clarithromycin
Uncomplicated community-acquired pneumonia Amoxicillin (Doxycycline or clarithromycin in penicillin allergic, add flucloxacillin if staphylococci suspected e.g. In influenza)
Pneumonia possibly caused by atypical pathogens Clarithromycin
Hospital-acquired pneumonia Within 5 days of admission: co-amoxiclav or cefuroxime
More than 5 days after admission: piperacillin with tazobactam OR a broad-spectrum cephalosporin (e.g. ceftazidime) OR a quinolone (e.g. ciprofloxacin)

Urinary tract

Condition Recommended treatment
Lower urinary tract infection Trimethoprim or nitrofurantoin. Alternative: amoxicillin or cephalosporin
Acute pyelonephritis Broad-spectrum cephalosporin or quinolone
Acute prostatitis Quinolone or trimethoprim

Skin

Condition Recommended treatment
Impetigo Topical hydrogen peroxide, oral flucloxacillin or erythromycin if widespread
Cellulitis Flucloxacillin (clarithromycin, erythromycin or doxycycline if penicillin-allergic)
Cellulitis (near the eyes or nose) Co-amoxiclav (clarithromycin, + metronidazole if penicillin-allergic)
Erysipelas Flucloxacillin* (clarithromycin, erythromycin or doxycycline if penicillin-allergic)
Animal or human bite Co-amoxiclav (doxycycline + metronidazole if penicillin-allergic)
Mastitis during breast-feeding Flucloxacillin

Ear, nose & throat

Condition Recommended treatment
Throat infections Phenoxymethylpenicillin (erythromycin alone if penicillin-allergic)
Sinusitis Phenoxymethylpenicillin
Otitis media Amoxicillin (erythromycin if penicillin-allergic)
Otitis externa** Flucloxacillin (erythromycin if penicillin-allergic)
Periapical or periodontal abscess Amoxicillin
Gingivitis: acute necrotising ulcerative Metronidazole

Genital system
Condition Recommended treatment
Gonorrhoea Intramuscular ceftriaxone
Chlamydia Doxycycline or azithromycin
Pelvic inflammatory disease Oral ofloxacin + oral metronidazole or intramuscular ceftriaxone + oral doxycycline + oral metronidazole
Syphilis Benzathine benzylpenicillin or doxycycline or erythromycin
Bacterial vaginosis Oral or topical metronidazole or topical clindamycin

Gastrointestinal

Condition Recommended treatment
Clostridium difficile First episode: metronidazole
Second or subsequent episode of infection: vancomycin
Campylobacter enteritis Clarithromycin
Salmonella (non-typhoid) Ciprofloxacin
Shigellosis Ciprofloxacin

How well did you know this?
1
Not at all
2
3
4
5
Perfectly
58
Q

How is HSV 1 and 2 diagnosed and managed?

A

There are two strains of the herpes simplex virus (HSV) in humans: HSV-1 and HSV-2. Whilst it was previously thought HSV-1 accounted for oral lesions (cold sores) and HSV-2 for genital herpes it is now known there is considerable overlap

Features
painful genital ulceration
may be associated with dysuria and pruritus
the primary infection is often more severe than recurrent episodes
systemic features such as headache, fever and malaise are more common in primary episodes
tender inguinal lymphadenopathy
urinary retention may occur

Investigations
nucleic acid amplification tests (NAAT) is the investigation of choice in genital herpes and are now considered superior to viral culture
HSV serology may be useful in certain situations such as recurrent genital ulceration of unknown cause

Management
general measures include:
saline bathing
analgesia
topical anaesthetic agents e.g. lidocaine
oral aciclovir
some patients with frequent exacerbations may benefit from longer-term aciclovir

Pregnancy
elective caesarean section at term is advised if a primary attack of herpes occurs during pregnancy at greater than 28 weeks gestation
women with recurrent herpes who are pregnant should be treated with suppressive therapy and be advised that the risk of transmission to their baby is low

Oral aciclovir 400 mg TDS (three times daily) until delivery is recommended in the RCOG guidelines for women who present with a primary herpes infection in their third trimester of pregnancy, especially if the woman is expected to deliver within 6 weeks.

How well did you know this?
1
Not at all
2
3
4
5
Perfectly
59
Q

How is UTI treated in men and pregnant women

A

Non-pregnant women
local antibiotic guidelines should be followed if available
CKS/2012 SIGN guidelines recommend trimethoprim or nitrofurantoin for 3 days
send a urine culture if:
aged > 65 years
visible or non-visible haematuria

Pregnant women
if the pregnant woman is symptomatic:
a urine culture should be sent in all cases
should be treated with an antibiotic for
first-line: nitrofurantoin (should be avoided near term)
second-line: amoxicillin or cefalexin
trimethoprim is teratogenic in the first trimester and should be avoided during pregnancy
asymptomatic bacteriuria in pregnant women:
a urine culture should be performed routinely at the first antenatal visit
Clinical Knowledge Summaries recommend an immediate antibiotic prescription of either nitrofurantoin (should be avoided near term), amoxicillin or cefalexin. This should be a 7-day course
the rationale of treating asymptomatic bacteriuria is the significant risk of progression to acute pyelonephritis
a further urine culture should be sent following completion of treatment as a test of cure

Men
an immediate antibiotic prescription should be offered for 7 days
as with non-pregnant women, trimethoprim or nitrofurantoin should be offered first-line unless prostatitis is suspected
NICE Clinical Knowledge Summaries state: ‘Referral to urology is not routinely required for men who have had one uncomplicated lower urinary tract infection (UTI).’

Catherised patients
do not treat asymptomatic bacteria in catheterised patients
if the patient is symptomatic they should be treated with an antibiotic
a 7-day, rather than a 3-day course should be given

Acute pyelonephritis

For patients with sign of acute pyelonephritis hospital admission should be considered
local antibiotic guidelines should be followed if available
the BNF currently recommends a broad-spectrum cephalosporin or a quinolone (for non-pregnant women) for 10-14 days

How well did you know this?
1
Not at all
2
3
4
5
Perfectly
60
Q

What organisms are patients with cystic fibrosis most at risk of getting?

A

Patients with cystic fibrosis develop bronchiectasis early on during their life resulting in repeated hospital admissions with lower respiratory tract infections. The pathological process behind bronchiectasis results in sputum pooling within the larger airways with poor removal. Subsequently colonisation occurs with bacteria and occasionally fungi. The most common bacteria is the gram negative rod Pseudomonas aeruginosa and should always be taken into account if providing empirical treatment. If the patient is systemically well then antibiotic sensitivities should be sought from a culture sample before starting treatment. However, an anti-pseudomonal agent such as piperacillin with tazobactam or ciprofloxacin should be used as part of empirical treatment for sepsis in cystic fibrosis patients.

How well did you know this?
1
Not at all
2
3
4
5
Perfectly
61
Q

What is Fitz Hugh Curtis syndrome and what conditions can cause this?

A

Fitz-Hugh-Curtis syndrome is a complication of pelvic inflammatory disease in which the liver capsule becomes inflamed causing right upper quadrant pain. This leads to scar tissue formation and peri-hepatic adhesions. It usually occurs in women who have either chlamydia or gonorrhoea.

Treatment is through eradication of the responsible organism although laparoscopy is required in some patients to perform lysis of adhesions that have formed.

How well did you know this?
1
Not at all
2
3
4
5
Perfectly
62
Q

How does leptospirosis present?How is it investigated and managed?

A

Leptospirosis is caused by the spirochaete Leptospira interrogans (serogroup L. icterohaemorrhagiae), classically being spread by contact with infected rat urine.

Epidemiology
leptospirosis is commonly seen in questions referring to sewage workers, farmers, vets or people who work in an abattoir
however, on an international level, leptospirosis is far more common in the tropics so should be considered in the returning traveller

Weil’s disease should always be considered in high-risk patients with hepatorenal failure

Features
the early phase is due to bacteraemia and lasts around a week
may be mild or subclinical
fever
flu-like symptoms
subconjunctival suffusion (redness)/haemorrhage
second immune phase may lead to more severe disease (Weil’s disease)
acute kidney injury (seen in 50% of patients)
hepatitis: jaundice, hepatomegaly
aseptic meningitis

Investigation
serology: antibodies to Leptospira develop after about 7 days
PCR
culture
growth may take several weeks so limits usefulness in diagnosis
blood and CSF samples are generally positive for the first 10 days
urine cultures become positive during the second week of illness

Management
high-dose benzylpenicillin or doxycycline

How well did you know this?
1
Not at all
2
3
4
5
Perfectly
63
Q

What are the causes of traveller’s diarrhoea and acute food poisoning

A

Gastroenteritis may either occur whilst at home or whilst travelling abroad (travellers’ diarrhoea)

Travellers’ diarrhoea may be defined as at least 3 loose to watery stools in 24 hours with or without one of more of abdominal cramps, fever, nausea, vomiting or blood in the stool. The most common cause is Escherichia coli.

Another pattern of illness is ‘acute food poisoning’. This describes the sudden onset of nausea, vomiting and diarrhoea after the ingestion of a toxin. Acute food poisoning is typically caused by Staphylococcus aureus, Bacillus cereus or Clostridium perfringens

How well did you know this?
1
Not at all
2
3
4
5
Perfectly
64
Q

How does botulism present?

A

Clostridium botulinum
gram positive anaerobic bacillus
7 serotypes A-G
produces botulinum toxin, a neurotoxin which irreversibly blocks the release of acetylcholine
may result from eating contaminated food (e.g. tinned) or intravenous drug use
neurotoxin often affects bulbar muscles and autonomic nervous system

Features
patient usually fully conscious with no sensory disturbance
flaccid paralysis
diplopia
ataxia
bulbar palsy

Treatment
botulism antitoxin and supportive care
antitoxin is only effective if given early - once toxin has bound its actions cannot be reversed

How well did you know this?
1
Not at all
2
3
4
5
Perfectly
65
Q

How does Mycoplasma pneumonia present and how is it investigated and managed?

A

Mycoplasma pneumoniae is a cause of atypical pneumonia which often affects younger patients. It is associated with a number of characteristic complications such as erythema multiforme and cold autoimmune haemolytic anaemia. Epidemics of Mycoplasma pneumoniae classically occur every 4 years. It is important to recognise atypical pneumonia as it may not respond to penicillins or cephalosporins due to it lacking a peptidoglycan cell wall.

Features
the disease typically has a prolonged and gradual onset
flu-like symptoms classically precede a dry cough
bilateral consolidation on x-ray
complications may occur as below

Complications
cold agglutins (IgM): may cause an haemolytic anaemia, thrombocytopenia
erythema multiforme, erythema nodosum
meningoencephalitis, Guillain-Barre syndrome and other immune-mediated neurological diseases
bullous myringitis: painful vesicles on the tympanic membrane
pericarditis/myocarditis
gastrointestinal: hepatitis, pancreatitis
renal: acute glomerulonephritis

Investigations
diagnosis is generally by Mycoplasma serology
positive cold agglutination test

Management
doxycycline or a macrolide (e.g. erythromycin/clarithromycin)

How well did you know this?
1
Not at all
2
3
4
5
Perfectly
66
Q

HIV drugs revision

A

Antiretroviral therapy (ART) involves a combination of at least three drugs, typically two nucleoside reverse transcriptase inhibitors (NRTI) and either a protease inhibitor (PI) or a non-nucleoside reverse transcriptase inhibitor (NNRTI). This combination both decreases viral replication but also reduces the risk of viral resistance emerging

Following the 2015 BHIVA guidelines it is now recommended that patients start ART as soon as they have been diagnosed with HIV, rather than waiting until a particular CD4 count, as was previously advocated.

Entry inhibitors
maraviroc (binds to CCR5, preventing an interaction with gp41), enfuvirtide (binds to gp41, also known as a ‘fusion inhibitor’)
prevent HIV-1 from entering and infecting immune cells

Nucleoside analogue reverse transcriptase inhibitors (NRTI)
examples: zidovudine (AZT), abacavir, emtricitabine, didanosine, lamivudine, stavudine, zalcitabine, tenofovir
general NRTI side-effects: peripheral neuropathy
tenofovir: used in BHIVAs two recommended regime NRTI. Adverse effects include renal impairment and ostesoporosis
zidovudine: anaemia, myopathy, black nails
didanosine: pancreatitis

Non-nucleoside reverse transcriptase inhibitors (NNRTI)
examples: nevirapine, efavirenz
side-effects: P450 enzyme interaction (nevirapine induces), rashes

Protease inhibitors (PI)
examples: indinavir, nelfinavir, ritonavir, saquinavir
side-effects: diabetes, hyperlipidaemia, buffalo hump, central obesity, P450 enzyme inhibition
indinavir: renal stones, asymptomatic hyperbilirubinaemia
ritonavir: a potent inhibitor of the P450 system

Integrase inhibitors
block the action of integrase, a viral enzyme that inserts the viral genome into the DNA of the host cell
examples: raltegravir, elvitegravir, dolutegravir

‘Navir tease a pro’: HIV drugs that end with -navir are protease inhibitors e.g. ritonavir. Note that due to tolerability issues, ritonavir is now more commonly used to boost the levels of other HIV drugs

‘It’s grave/great you integrate’: HIV drugs that end with -gravir are integrase inhibitors e.g. raltegravir

How well did you know this?
1
Not at all
2
3
4
5
Perfectly
67
Q

What to do if patient test negative for HIV on first test?should test be repeated and if so after hoe long?

A

HIV seroconversion occurs from 3-12 weeks
Important for meLess important
Seroconversion, the period where an antibody response is created and is detectable, can occur between 3 to 12 weeks. Most people create antibodies between 4 and 6 weeks. However, even if patients take the ELISA test and receive a negative result, they are recommended to redo the test 3 months (after exposure) to confirm they are HIV free.

How well did you know this?
1
Not at all
2
3
4
5
Perfectly
68
Q

What prophylaxis should be given to patients with HIV and why?

A

All patients with a CD4 count lower than 200/mm3 should receive prophylaxis against Pneumocystis jiroveci pneumonia
Important for meLess important
All patients with a CD4 count lower than 200 cells/mm³ should receive prophylaxis against Pneumocystis jirovecii pneumonia.

Aciclovir is sometimes used as longterm herpes suppression treatment in patients with both HIV and HSV.

How well did you know this?
1
Not at all
2
3
4
5
Perfectly
69
Q

How do treated cellulitis in pregnancy?

A

The BNF recommends flucloxacillin as first-line treatment for mild/moderate cellulitis. Clarithromycin, erythromycin (in pregnancy) or doxycyline is recommended in patients allergic to penicillin.

NICE recommend that patients severe cellulitis should be offered co-amoxiclav, cefuroxime, clindamycin or ceftriaxone.

How well did you know this?
1
Not at all
2
3
4
5
Perfectly
70
Q

How can gonorrhoea be treated?

A

For patients with gonorrhoea, a combination of oral cefixime + oral azithromycin is used if the patient refuses IM ceftriaxone
Important for meLess important
NICE guidelines advise that patients with a positive gonorrhoea test should be referred to a sexual health clinic for management and follow up.

The first-line treatment is a single dose of IM ceftriaxone.

How well did you know this?
1
Not at all
2
3
4
5
Perfectly
71
Q

What are the features of disseminated gonnorhea?

A

Disseminated gonococcal infection (DGI) and gonococcal arthritis may also occur, with gonococcal infection being the most common cause of septic arthritis in young adults. The pathophysiology of DGI is not fully understood but is thought to be due to haematogenous spread from mucosal infection (e.g. Asymptomatic genital infection). Initially there may be a classic triad of symptoms: tenosynovitis, migratory polyarthritis and dermatitis. Later complications include septic arthritis, endocarditis and perihepatitis (Fitz-Hugh-Curtis syndrome)

Key features of disseminated gonococcal infection
tenosynovitis
migratory polyarthritis
dermatitis (lesions can be maculopapular or vesicular)

72
Q

What vaccinations are routinely offered to pregnant women in the UK?

A

Influenza and pertussis

73
Q

How does yellow fever present?

A

Type of viral haemorrhagic fever (also dengue fever, Lassa fever, Ebola).

Basics
zoonotic infection: spread by Aedes mosquitos
incubation period = 2 - 14 days

Features
may cause mild flu-like illness lasting less than one week
classic description involves sudden onset of high fever, rigors, nausea & vomiting. Bradycardia may develop. A brief remission is followed by jaundice, haematemesis, oliguria
if severe jaundice, haematemesis may occur
Councilman bodies (inclusion bodies) may be seen in the hepatocytes

The pattern of disease is most consistent with yellow fever. Classically it will present in two phases where the patient experiences a brief remission in between. Yellow fever is mostly concentrated in Africa but it still persists in some rural areas of South America. It can present very quickly with non-specific symptoms and it has an incubation period of 2-14 days which is fitting with this patients history.

74
Q

How care contacts screened for latent TB?

A

Screening for latent tuberculosis

The Mantoux test is the main technique used to screen for latent tuberculosis. In recent years the interferon-gamma blood test has also been introduced. It is used in a number of specific situations such as:
the Mantoux test is positive or equivocal
people where a tuberculin test may be falsely negative (see below)

Mantoux test
0.1 ml of 1:1,000 purified protein derivative (PPD) injected intradermally
result read 2-3 days later

Diameter of induration Positivity Interpretation
< 6mm Negative - no significant hypersensitivity to tuberculin protein Previously unvaccinated individuals may be given the BCG
6 - 15mm Positive - hypersensitive to tuberculin protein Should not be given BCG. May be due to previous TB infection or BCG
> 15mm Strongly positive - strongly hypersensitive to tuberculin protein Suggests tuberculosis infection.

False negative tests may be caused by:
miliary TB
sarcoidosis
HIV
lymphoma
very young age (e.g. < 6 months)
75
Q

How are patients screened for active TB?

A

Chest x-ray
upper lobe cavitation is the classical finding of reactivated TB
bilateral hilar lymphadenopathy

Sputum smear
3 specimens are needed
rapid and inexpensive test
stained for the presence of acid-fast bacilli (Ziehl-Neelsen stain)
all mycobacteria will stain positive (i.e. nontuberculous mycobacteria)
the sensitivity is between 50-80%
this is decreased in individuals with HIV to around 20-30%

Sputum culture
the gold standard investigation
more sensitive than a sputum smear and nucleic acid amplification tests
can assess drug sensitivities
can take 1-3 weeks (if using liquid media, longer if solid media)

Nucleic acid amplification tests (NAAT)
allows rapid diagnosis (within 24-48 hours)
more sensitive than smear but less sensitive than culture

76
Q

When is Men ACWY vaccine given

A

Both the meningitis ACWY and tetanus, diphtheria and polio vaccines are given at 14 years (school year 9) so these options are incorrect.

77
Q

What is. PML?

A

Progressive multifocal leukoencephalopathy (PML)
widespread demyelination
due to infection of oligodendrocytes by JC virus (a polyoma DNA virus)
symptoms, subacute onset : behavioural changes, speech, motor, visual impairment
CT: single or multiple lesions, no mass effect, don’t usually enhance. MRI is better - high-signal demyelinating white matter lesions are seen

78
Q

How is chronic hep C treated?what ate the complications

A

Chronic hepatitis C may be defined as the persistence of HCV RNA in the blood for 6 months.

Potential complications of chronic hepatitis C
rheumatological problems: arthralgia, arthritis
eye problems: Sjogren’s syndrome
cirrhosis (5-20% of those with chronic disease)
hepatocellular cancer
cryoglobulinaemia: typically type II (mixed monoclonal and polyclonal)
porphyria cutanea tarda (PCT): it is increasingly recognised that PCT may develop in patients with hepatitis C, especially if there are other factors such as alcohol abuse
membranoproliferative glomerulonephritis

Management of chronic infection
treatment depends on the viral genotype - this should be tested prior to treatment
the management of hepatitis C has advanced rapidly in recent years resulting in clearance rates of around 95%. Interferon based treatments are no longer recommended
the aim of treatment is sustained virological response (SVR), defined as undetectable serum HCV RNA six months after the end of therapy
currently a combination of protease inhibitors (e.g. daclatasvir + sofosbuvir or sofosbuvir + simeprevir) with or without ribavirin are used

Complications of treatment
ribavirin - side-effects: haemolytic anaemia, cough. Women should not become pregnant within 6 months of stopping ribavirin as it is teratogenic
interferon alpha - side-effects: flu-like symptoms, depression, fatigue, leukopenia, thrombocytopeni

79
Q

What Infection are renal transplant patients at most risk of?

A

Over 50% of renal transplant patients have a significant infection within the first 12 months of having a renal transplant.

At the time of transplant the CMV-serological status of the donor and recipient are noted. The highest risk is seen in CMV-seronegative recipients who receive a kidney from a CMV-seropositive donor. These patients are usually given antiviral prophylaxis.

Cytomegalovirus tend to be seen after four weeks as before this time the immune system has not been fully affected by the immunosuppressants.

80
Q

When should antibiotics be given pre operatively?

A

Preoperatively
Don’t remove body hair routinely
If hair needs removal, use electrical clippers with single use head (razors increase infection risk)
Antibiotic prophylaxis if:
- placement of prosthesis or valve
- clean-contaminated surgery
- contaminated surgery
Use local formulary
Aim to give single dose IV antibiotic on anaesthesia
If a tourniquet is to be used, give prophylactic antibiotics earlier

81
Q

How can toxoplasmosis be caused?

A

mmunocompotent patients

Most infections are asymptomatic. Symptomatic patients usually have a self-limiting infection, often having clinical features resembling infectious mononucleosis (fever, malaise, lymphadenopathy). Other less common manifestations include meningoencephalitis and myocarditis.

Serology is the investigation of choice.

No treatment is usually required unless the patient has a severe infection or is immunosuppressed.

HIV/immunosuppressed patients

Cerebral toxoplasmosis accounts for around 50% of cerebral lesions in patients with HIV
constitutional symptoms, headache, confusion, drowsiness
CT: usually single or multiple ring-enhancing lesions, mass effect may be seen
management: pyrimethamine plus sulphadiazine for at least 6 weeks

Immunosuppressed patients may also develop a chorioretinitis secondary to toxoplasmosis.

Congenital toxoplasmosis

Congenital toxoplasmosis is due to transplacental spread from the mother. It causes a variety of effects to the unborn child including
neurological damage
cerebral calcification
hydrocephalus
chorioretinitis
ophthalmic damage
retinopathy
cataracts
82
Q

Which Micro organism infection can cause reactivation of HSV

A

Streptococcus pneumoniae commonly causes reactivation of the herpes simplex virus resulting in ‘cold sores’

Discuss (4)
Improve

83
Q

How would TCA overdose present?

A

Overdose of tricyclic antidepressants is a common presentation to emergency departments. Amitriptyline and dosulepin (dothiepin) are particularly dangerous in overdose.

Early features relate to anticholinergic properties: dry mouth, dilated pupils, agitation, sinus tachycardia, blurred vision.

Features of severe poisoning include:
arrhythmias
seizures
metabolic acidosis
coma

ECG changes include:
sinus tachycardia
widening of QRS
prolongation of QT interval

Widening of QRS > 100ms is associated with an increased risk of seizures whilst QRS > 160ms is associated with ventricular arrhythmias

Management
IV bicarbonate
first-line therapy for hypotension or arrhythmias
indications include widening of the QRS interval >100 msec or a ventricular arrhythmia
other drugs for arrhythmias
+ class 1a (e.g. Quinidine) and class Ic antiarrhythmics (e.g. Flecainide) are contraindicated as they prolong depolarisation
class III drugs such as amiodarone should also be avoided as they prolong the QT interval
response to lignocaine is variable and it should be emphasized that correction of acidosis is the first line in management of tricyclic induced arrhythmias
intravenous lipid emulsion is increasingly used to bind free drug and reduce toxicity
dialysis is ineffective in removing tricyclics

84
Q

What are the risk factors for paracetamol overdose

A
The following groups of patients are at an increased risk of developing hepatotoxicity following a paracetamol overdose:
patients taking liver enzyme-inducing drugs (rifampicin, phenytoin, carbamazepine, chronic alcohol excess, St John's Wort)
malnourished patients (e.g. anorexia nervosa) or patients who have not eaten for a few days

Interestingly, acute alcohol intake, as opposed to chronic alcohol excess, is not associated with an increased risk of developing hepatotoxicity and may actually be protective.

Paracetamol overdose - high risk if chronic alcohol, HIV, anorexia or P450 inducers

85
Q

What drugs can cause an oculogyric crisis?

A

An oculogyric crisis is a dystonic reaction to certain drugs or medical conditions

Features
restlessness, agitation
involuntary upward deviation of the eyes

Causes
antipsychotics
metoclopramide
postencephalitic Parkinson’s disease

Management
intravenous antimuscarinic: benztropine or procyclidine

86
Q

How does opioid misuse present?what are the complication

A

Opioids are substances which bind to opioid receptors. This includes both naturally occurring opiates such as morphine and synthetic opioids such as buprenorphine and methadone.

Features of opioid misuse
rhinorrhoea
needle track marks
pinpoint pupils
drowsiness
watering eyes
yawning

Complications of opioid misuse
viral infection secondary to sharing needles: HIV, hepatitis B & C
bacterial infection secondary to injection: infective endocarditis, septic arthritis, septicaemia, necrotising fasciitis
venous thromboembolism
overdose may lead to respiratory depression and death
psychological problems: craving
social problems: crime, prostitution, homelessness

87
Q

How can opioid misuse be treated?What detox drugs are used

A

Emergency management of opioid overdose
IV or IM naloxone: has a rapid onset and relatively short duration of action

Harm reduction interventions may include
needle exchange
offering testing for HIV, hepatitis B & C

Management of opioid dependence
patients are usually managed by specialist drug dependence clinics although some GPs with a specialist interest offer similar services
patients may be offered maintenance therapy or detoxification
NICE recommend methadone or buprenorphine as the first-line treatment in opioid detoxification
compliance is monitored using urinalysis
detoxification should normally last up to 4 weeks in an inpatient/residential setting and up to 12 weeks in the community

88
Q

How does LSD intoxication present? How is it managed?

A

Lysergic acid diethylamide (LSD) is the first synthetic hallucinogen which gained popularity in the 1960s to 1980s as a recreational drug. Its psychedelic effects usually involve heightening or distortion of sensory stimuli and enhancement of feelings and introspection. It is one of the most potent psychoactive compounds known.

Epidemiology
Its usage has declined in recent years but still persists.
The Drug Abuse Warning Network (DAWN) estimated that in 2011, the incidence of LSD-related emergency department attendance in the U.S. is 1.5 per 100,000 population.
Adolescents and young adults are the most frequent users.

Patients with LSD toxicity typically present following acute panic reactions (known as bad trips), massive ingestions or unintentional ingestions.

Psychoactive symptoms
Variable subjective experiences
Impaired judgements which can lead to injury
Amplification of current mood which leads to euphoria or dysphoria
Agitation, appearing withdrawn - especially in inexperienced users
Drug-induced psychosis

Somatic symptoms
Nausea
Headache
Palpitations
Dry mouth
Drowsiness
Tremors
Signs
Tachycardia
Hypertension
Mydriasis
Paresthesia
Hyperreflexia
Pyrexia

Manifestations such as tachycardia, hypertension, pupillary dilation, tremor, and hyperpyrexia can occur within minutes following oral administration of 0.5–2 µg/kg.

Massive overdoses can lead to the following complications:
Respiratory arrest
Coma
Hyperthermia
Autonomic dysfunction
Bleeding disorders

Investigations:
The diagnosis of LSD toxicity is mainly based on history and examination.
Most urine drug screens do not pick up LSD.

Management
Management of the intoxicated patient is dependent on the specific behavioural manifestation elicited by the drug.
Agitation, e.g. from a ‘bad trip’, should be first managed with supportive reassurance in a calm, stress-free environment. If ineffective, benzodiazepines are the medication of choice.
LSD-induced psychosis may require antipsychotics.
Massive ingestions of LSD should be treated with supportive care, including respiratory support and endotracheal intubation if needed. Hypertension, tachycardia, and hyperthermia should be treated symptomatically. Hypotension should be treated initially with fluids and subsequently with vasopressors if required.
Because LSD is rapidly absorbed through the gastrointestinal tract, activated charcoal administration and gastric emptying are of little clinical value by the time a patient presents to the emergency department.

89
Q

What are the contraindication for metformin and when should it be withdrawn?(surgery)

A

Contraindications
chronic kidney disease: NICE recommend that the dose should be reviewed if the creatinine is > 130 µmol/l (or eGFR < 45 ml/min) and stopped if the creatinine is > 150 µmol/l (or eGFR < 30 ml/min)
metformin may cause lactic acidosis if taken during a period where there is tissue hypoxia. Examples include a recent myocardial infarction, sepsis, acute kidney injury and severe dehydration
iodine-containing x-ray contrast media: examples include peripheral arterial angiography, coronary angiography, intravenous pyelography (IVP); there is an increasing risk of provoking renal impairment due to contrast nephropathy; metformin should be discontinued on the day of the procedure and for 48 hours thereafter
alcohol abuse is a relative contraindication

Starting metformin
metformin should be titrated up slowly to reduce the incidence of gastrointestinal side-effects
if patients develop unacceptable side-effects then modified-release metformin should be considered

90
Q

What metabolic abnormality is seen in salicylate overdose and how can overdose present and be managed?

A

Salicylate overdose

A key concept for the exam is to understand that salicylate overdose leads to a mixed respiratory alkalosis and metabolic acidosis. Early stimulation of the respiratory centre leads to a respiratory alkalosis whilst later the direct acid effects of salicylates (combined with acute renal failure) may lead to an acidosis. In children metabolic acidosis tends to predominate.

Features
hyperventilation (centrally stimulates respiration)
tinnitus
lethargy
sweating, pyrexia*
nausea/vomiting
hyperglycaemia and hypoglycaemia
seizures
coma

Treatment
general (ABC, charcoal)
urinary alkalinization with intravenous sodium bicarbonate - enhances elimination of aspirin in the urine
haemodialysis

Indications for haemodialysis in salicylate overdose
serum concentration > 700mg/L
metabolic acidosis resistant to treatment
acute renal failure
pulmonary oedema
seizures
coma

*salicylates cause the uncoupling of oxidative phosphorylation leading to decreased adenosine triphosphate production, increased oxygen consumption and increased carbon dioxide and heat production

91
Q

How does lithium toxicity present and how is it managed?

A

Lithium is a mood stabilising drug used most commonly prophylactically in bipolar disorder but also as an adjunct in refractory depression. It has a very narrow therapeutic range (0.4-1.0 mmol/L) and a long plasma half-life being excreted primarily by the kidneys. Lithium toxicity generally occurs following concentrations > 1.5 mmol/L.

Toxicity may be precipitated by:
dehydration
renal failure
drugs: diuretics (especially thiazides), ACE inhibitors/angiotensin II receptor blockers, NSAIDs and metronidazole.

Features of toxicity
coarse tremor (a fine tremor is seen in therapeutic levels)
hyperreflexia
acute confusion
polyuria
seizure
coma

Management
mild-moderate toxicity may respond to volume resuscitation with normal saline
haemodialysis may be needed in severe toxicity
sodium bicarbonate is sometimes used but there is limited evidence to support this. By increasing the alkalinity of the urine it promotes lithium excretion

92
Q

How can common overdoses be managed?(antidote

A

ParacetamolManagement

  • activated charcoal if ingested < 1 hour ago
  • N-acetylcysteine (NAC)
  • liver transplantation

SalicylateManagement

• urinary alkalinization with IV bicarbonate
• haemodialysis
Opioid/opiatesNaloxoneBenzodiazepinesFlumazenil
The majority of overdoses are managed with supportive care only due to the risk of seizures with flumazenil. It is generally only used with severe or iatrogenic overdoses.

Tricyclic antidepressantsManagement

  • IV bicarbonate may reduce the risk of seizures and arrhythmias in severe toxicity
  • arrhythmias: class 1a (e.g. Quinidine) and class Ic antiarrhythmics (e.g. Flecainide) are contraindicated as they prolong depolarisation. Class III drugs such as amiodarone should also be avoided as they prolong the QT interval. Response to lignocaine is variable and it should be emphasized that correction of acidosis is the first line in management of tricyclic induced arrhythmias
  • dialysis is ineffective in removing tricyclics

LithiumManagement

• mild-moderate toxicity may respond to volume resuscitation with normal saline
• haemodialysis may be needed in severe toxicity
• sodium bicarbonate is sometimes used but there is limited evidence to support this. By increasing the alkalinity of the urine it promotes lithium excretion
WarfarinVitamin K, prothrombin complexHeparinProtamine sulphate

Beta-blockersManagement

  • if bradycardic then atropine
  • in resistant cases glucagon may be used

Ethylene glycolManagement has changed in recent times

  • ethanol has been used for many years
  • works by competing with ethylene glycol for the enzyme alcohol dehydrogenase
  • this limits the formation of toxic metabolites (e.g. Glycoaldehyde and glycolic acid) which are responsible for the haemodynamic/metabolic features of poisoning
  • fomepizole, an inhibitor of alcohol dehydrogenase, is now used first-line in preference to ethanol
  • haemodialysis also has a role in refractory cases

Methanol poisoningManagement

  • fomepizole or ethanol
  • haemodialysis

Organophosphate insecticidesManagement

  • atropine
  • the role of pralidoxime is still unclear - meta-analyses to date have failed to show any clear benefit

DigoxinDigoxin-specific antibody fragments

IronDesferrioxamine, a chelating agent
LeadDimercaprol, calcium edetate

Carbon monoxideManagement

  • 100% oxygen
  • hyperbaric oxygen

CyanideHydroxocobalamin; also combination of amyl nitrite, sodium nitrite, and sodium thiosulfate

93
Q

How can alcoholic patients be managed?

A

Ethanol reduces the calcium-dependent secretion of anti-diuretic hormone (ADH) by blocking channels in the neurohypophyseal nerve terminal.

Nausea associated with hangovers is mainly due to vagal stimulation to the vomiting centre. Following a particular severe episode of alcohol excess people may experience tremors. These are due to increased glutamate production by neurones to compensate for the previous inhibition by ethanol.

Nutritional support
SIGN recommends alcoholic patients should receive oral thiamine if their ‘diet may be deficient’

Drugs used
benzodiazepines for acute withdrawal
disulfram: promotes abstinence - alcohol intake causes severe reaction due to inhibition of acetaldehyde dehydrogenase. Patients should be aware that even small amounts of alcohol (e.g. In perfumes, foods, mouthwashes) can produce severe symptoms. Contraindications include ischaemic heart disease and psychosis
acamprosate: reduces craving, known to be a weak antagonist of NMDA receptors, improves abstinence in placebo controlled trials

94
Q

How can serotonin

A
Causes
monoamine oxidase inhibitors
SSRIs
St John's Wort, often taken over the counter for depression, can interact with SSRIs to cause serotonin syndrome
ecstasy
amphetamines

Features
neuromuscular excitation (e.g. hyperreflexia, myoclonus, rigidity)
autonomic nervous system excitation (e.g. hyperthermia)
altered mental state

Management
supportive including IV fluids
benzodiazepines
more severe cases are managed using serotonin antagonists such as cyproheptadine and chlorpromazine

95
Q

What are the common side effects of antihypertensive drugs?

A
Drug	Side-effect
ACE inhibitors	• Cough
• Hyperkalaemia
Bendroflumethiazide	• Gout
• Hypokalaemia
• Hyponatraemia
• Impaired glucose tolerance
Calcium channel blockers	• Headache
• Flushing
• Ankle oedema
Beta-blockers	• Bronchospasm (especially in asthmatics)
• Fatigue
• Cold peripheries
Doxazosin	• Postural hypotension
96
Q

Drug monitoring for common drugs

A

The tables below show the monitoring requirements of common drugs. It should be noted these are basic guidelines and do not relate to monitoring effectiveness of treatment (e.g. Checking lipids for patients taking a statin)

Cardiovascular drugs

Drug Main monitoring parameters Details of monitoring
Statins LFT LFTs at baseline, 3 months and 12 months
ACE inhibitors U&E U&E prior to treatment
U&E after increasing dose
U&E at least annually
Amiodarone TFT, LFT TFT, LFT, U&E, CXR prior to treatment
TFT, LFT every 6 months

Rheumatology drugs

Drug Main monitoring parameters Details of monitoring
Methotrexate FBC, LFT, U&E The Committee on Safety of Medicines recommend ‘FBC and renal and LFTs before starting treatment and repeated weekly until therapy stabilised, thereafter patients should be monitored every 2-3 months’
Azathioprine FBC, LFT FBC, LFT before treatment
FBC weekly for the first 4 weeks
FBC, LFT every 3 months

Neuropsychiatric drugs

Drug Main monitoring parameters Details of monitoring
Lithium Lithium level, TFT, U&E TFT, U&E prior to treatment
Lithium levels weekly until stabilised then every 3 months
TFT, U&E every 6 months
Sodium valproate LFT LFT, FBC before treatment
LFT ‘periodically’ during first 6 months

Endocrine drugs

Drug Main monitoring parameters Details of monitoring
Glitazones LFT LFT before treatment
LFT ‘regularly’ during treatment

97
Q

How does cocaine intoxication present

A

Cocaine is an alkaloid derived from the coca plant. It is widely used as a recreational stimulant. The price of cocaine has fallen sharply in the past decade resulting in cocaine toxicity becoming a much more frequent clinical problem. This increase has made cocaine a favourite topic of question writers.

Mechanism of action
cocaine blocks the uptake of dopamine, noradrenaline and serotonin

The use of cocaine is associated with a wide variety of adverse effects:

Cardiovascular effects
myocardial infarction
both tachycardia and bradycardia may occur
hypertension
QRS widening and QT prolongation
aortic dissection
Neurological effects
seizures
mydriasis
hypertonia
hyperreflexia

Psychiatric effects
agitation
psychosis
hallucinations

Others
ischaemic colitis is recognised in patients following cocaine ingestion. This should be considered if patients complain of abdominal pain or rectal bleeding
hyperthermia
metabolic acidosis
rhabdomyolysis

Management of cocaine toxicity
in general, benzodiazepines are generally first-line for most cocaine-related problems
chest pain: benzodiazepines + glyceryl trinitrate. If myocardial infarction develops then primary percutaneous coronary intervention
hypertension: benzodiazepines + sodium nitroprusside
the use of beta-blockers in cocaine-induced cardiovascular problems is a controversial issue. The American Heart Association issued a statement in 2008 warning against the use of beta-blockers (due to the risk of unopposed alpha-mediated coronary vasospasm) but many cardiologists since have questioned whether this is valid. If a reasonable alternative is given in an exam it is probably wise to choose it

98
Q

What are the side effects of mefloquine?

A

Mefloquine (brand name Lariam) is used for both the prophylaxis and treatment of certain types of malaria. There has long been a concern about the neuropsychiatric side-effects of mefloquine. A recent review has however led to ‘strengthened warnings’ about the potential risks.

The following advice is therefore given:
certain side-effects such nightmares or anxiety may be ‘prodromal’ of a more serious neuropsychiatric event
suicide and deliberate self harm have been reported in patients taking mefloquine
adverse reactions may continue for several months due to the long half-life or mefloquine
mefloquine should not be used in patients with a history of anxiety, depression schizophrenia or other psychiatric disorders
patients who experience neuropsychiatric sife-effects should stop mefloquine and seek medical advice

99
Q

What are the clinical features of digoxin toxicity?

A

Clinical features of digoxin toxicity*:
GIT: nausea, vomiting, anorexia, diarrhoea
Visual: blurred vision, yellow/green discolouration, haloes
CVS: palpitations, syncope, dyspnoea
CNS: confusion, dizziness, delirium, fatigue

Management
Digibind
correct arrhythmias
monitor potassium

100
Q

When is Acetyl cysteine given in PCM overdose?What are the side effects seen?

A

Acetylcysteine should be given if:
there is a staggered overdose* or there is doubt over the time of paracetamol ingestion, regardless of the plasma paracetamol concentration; or
the plasma paracetamol concentration is on or above a single treatment line joining points of 100 mg/L at 4 hours and 15 mg/L at 15 hours, regardless of risk factors of hepatotoxicity

Acetylcysteine is now infused over 1 hour (rather than the previous 15 minutes) to reduce the number of adverse effects. Acetylcysteine commonly causes an anaphylactoid reaction (non-IgE mediated mast cell release). Anaphylactoid reactions to IV acetylcysteine are generally treated by stopping the infusion, then restarting at a slower rate.

Acetylcysteine should be given if:
there is a staggered overdose* or there is doubt over the time of paracetamol ingestion, regardless of the plasma paracetamol concentration; or
the plasma paracetamol concentration is on or above a single treatment line joining points of 100 mg/L at 4 hours and 15 mg/L at 15 hours, regardless of risk factors of hepatotoxicity

Acetylcysteine is now infused over 1 hour (rather than the previous 15 minutes) to reduce the number of adverse effects. Acetylcysteine commonly causes an anaphylactoid reaction (non-IgE mediated mast cell release). Anaphylactoid reactions to IV acetylcysteine are generally treated by stopping the infusion, then restarting at a slower rate.

101
Q

What is the king’s college criteria for liver transplantation?

A

Acetylcysteine should be given if:
there is a staggered overdose* or there is doubt over the time of paracetamol ingestion, regardless of the plasma paracetamol concentration; or
the plasma paracetamol concentration is on or above a single treatment line joining points of 100 mg/L at 4 hours and 15 mg/L at 15 hours, regardless of risk factors of hepatotoxicity

Acetylcysteine is now infused over 1 hour (rather than the previous 15 minutes) to reduce the number of adverse effects. Acetylcysteine commonly causes an anaphylactoid reaction (non-IgE mediated mast cell release). Anaphylactoid reactions to IV acetylcysteine are generally treated by stopping the infusion, then restarting at a slower rate.

102
Q

What are the adverse reaction and contraindications of quinolones like ciprofloxacin

A

Adverse effects
lower seizure threshold in patients with epilepsy
tendon damage (including rupture) - the risk is increased in patients also taking steroids
cartilage damage has been demonstrated in animal models and for this reason quinolones are generally avoided (but not necessarily contraindicated) in children
lengthens QT interval

Contraindications
Quinolones should generally be avoided in women who are pregnant or breastfeeding
avoid in G6PD

103
Q

Drugs causing ocular problems?

A

Cataracts
steroids

Corneal opacities
amiodarone
indomethacin

Optic neuritis
ethambutol
amiodarone
metronidazole

Retinopathy
chloroquine, quinine

Sildenafil can cause both blue discolouration and non-arteritic anterior ischaemic neuropathy

104
Q

How does organophosphate poisoning present and how is it managed?

A

One of the effects of organophosphate poisoning is inhibition of acetylcholinesterase leading to upregulation of nicotinic and muscarinic cholinergic neurotransmission. In warfare, sarin gas is a highly toxic synthetic organophosphorus compound that has similar effects.

Features can be predicted by the accumulation of acetylcholine (mnemonic = SLUD) 
Salivation
Lacrimation
Urination
Defecation/diarrhoea
cardiovascular: hypotension, bradycardia
also: small pupils, muscle fasciculation

Management
atropine
the role of pralidoxime is still unclear - meta-analyses to date have failed to show any clear benefit

105
Q

How can rheumatoid disease be monitored?

A

NICE recommends using a combination of CRP and disease activity (using a composite score such as DAS28) to assess response to treatment

106
Q

What test need to be done for methotrexate and why?

A
DMARDs
methotrexate is the most widely used DMARD. Monitoring of FBC & LFTs is essential due to the risk of myelosuppression and liver cirrhosis. Other important side-effects include pneumonitis
sulfasalazine
leflunomide
hydroxychloroquine

TNF-inhibitors
the current indication for a TNF-inhibitor is an inadequate response to at least two DMARDs including methotrexate
etanercept: recombinant human protein, acts as a decoy receptor for TNF-α, subcutaneous administration, can cause demyelination, risks include reactivation of tuberculosis
infliximab: monoclonal antibody, binds to TNF-α and prevents it from binding with TNF receptors, intravenous administration, risks include reactivation of tuberculosis
adalimumab: monoclonal antibody, subcutaneous administration

Rituximab
anti-CD20 monoclonal antibody, results in B-cell depletion
two 1g intravenous infusions are given two weeks apart
infusion reactions are common

Abatacept
fusion protein that modulates a key signal required for activation of T lymphocytes
leads to decreased T-cell proliferation and cytokine production
given as an infusion
not currently recommend by NICE

107
Q

How does polynyaliga present?How is it diagnosed and treated?

A

Polymyalgia rheumatica (PMR) is a relatively common condition seen in older people characterised by muscle stiffness and raised inflammatory markers. Whilst it appears to be closely related to temporal arteritis the underlying cause is not fully understood and it does not appear to be a vasculitic process.

Features
typically patient > 60 years old
usually rapid onset (e.g. < 1 month)
aching, morning stiffness in proximal limb muscles
weakness is not considered a symptom of polymyalgia rheumatica
also mild polyarthralgia, lethargy, depression, low-grade fever, anorexia, night sweats

Investigations
raised inflammatory markers e.g. ESR > 40 mm/hr
note creatine kinase and EMG normal

Treatment
prednisolone e.g. 15mg/od
patients typically respond dramatically to steroids, failure to do so should prompt consideration of an alternative diagnosis

108
Q

What are the side effects of hydroxychloroquine and how is it monitored?

A

Hydroxychloroquine is used in the management of rheumatoid arthritis and systemic/discoid lupus erythematosus. It is pharmacologically very similar to chloroquine which is used to treat certain types of malaria.

Adverse effects
bull’s eye retinopathy - may result in severe and permanent visual loss
recent data suggest that retinopathy caused by hydroxychloroquine is more common than previously thought and the most recent RCOphth guidelines (March 2018) suggest colour retinal photography and spectral domain optical coherence tomography scanning of the macula
baseline ophthalmological examination and annual screening is generally recommened

A contrast to many drugs used in rheumatology, hydroxychloroquine may be used if needed in pregnant women.

Monitoring
the BNF advises: ‘Ask patient about visual symptoms and monitor visual acuity annually using the standard reading chart’

109
Q

What Medications may worsen osteoporosis

A
Medications that may worsen osteoporosis (other than glucocorticoids):
SSRIs
antiepileptics
proton pump inhibitors
glitazones
long term heparin therapy
aromatase inhibitors e.g. anastrozole
110
Q

How can secondary causes of osteoporosis be investigated?

A

If a patient is diagnosed with osteoporosis or has a fragility fracture further investigations may be warranted. NOGG recommend testing for the following reasons:
exclude diseases that mimic osteoporosis (e.g. osteomalacia, myeloma);
identify the cause of osteoporosis and contributory factors;
assess the risk of subsequent fractures;
select the most appropriate form of treatment

The following investigations are recommended by NOGG:
History and physical examination
Blood cell count, sedimentation rate or C-reactive protein, serum calcium,
albumin, creatinine, phosphate, alkaline phosphatase and liver transaminases
Thyroid function tests
Bone densitometry ( DXA)

Other procedures, if indicated
Lateral radiographs of lumbar and thoracic spine/DXA-based vertebral imaging
Protein immunoelectrophoresis and urinary Bence-Jones proteins
25OHD
PTH
Serum testosterone, SHBG, FSH, LH (in men),
Serum prolactin
24 hour urinary cortisol/dexamethasone suppression test
Endomysial and/or tissue transglutaminase antibodies (coeliac disease)
Isotope bone scan
Markers of bone turnover, when available
Urinary calcium excretion

So from the first list we should order the following bloods as a minimum for all patients:
full blood count
urea and electrolytes
liver function tests
bone profile
CRP
thyroid function tests
111
Q

What is still’s disease and how does it present?

A

Epidemiology
has a bimodal age distribution - 15-25 yrs and 35-46 yrs

Features
arthralgia
elevated serum ferritin
rash: salmon-pink, maculopapular
pyrexia
typically rises in the late afternoon/early evening in a daily pattern and accompanies a worsening of joint symptoms and rash
lymphadenopathy
rheumatoid factor (RF) and anti-nuclear antibody (ANA) negative

The diagnosis of Still’s disease in adults can be challenging. The Yamaguchi criteria is the most widely used criteria and has a sensitivity of 93.5%.

Management
NSAIDs
should be used first-line to manage fever, joint pain and serositis
they should be trialled for at least a week before steroids are added.
steroids
may control symptoms but won’t improve prognosis
if symptoms persist, the use of methotrexate, IL-1 or anti-TNF therapy can be considered

112
Q

What Investigations need to be done before starting biologics

A

It is important to perform a chest X-ray to look for TB prior to starting biologics for rheumatoid arthritis as they can cause reactivation

113
Q

What are the roles and side effects of interferon alpha

A

Interferons (IFN) are cytokines released by the body in response to viral infections and neoplasia. They are classified according to cellular origin and the type of receptor they bind to. IFN-alpha and IFN-beta bind to type 1 receptors whilst IFN-gamma binds only to type 2 receptors.

Interferon-alpha
produced by leucocytes
antiviral action
useful in hepatitis B & C, Kaposi’s sarcoma, metastatic renal cell cancer, hairy cell leukaemia
adverse effects include flu-like symptoms and depression

Interferon-beta
produced by fibroblasts
antiviral action
reduces the frequency of exacerbations in patients with relapsing-remitting MS

Interferon-gamma
predominately natural killer cells. Also by T helper cells
weaker antiviral action, more of a role in immunomodulation particularly macrophage activation
may be useful in chronic granulomatous disease and osteopetrosis

114
Q

What are the complications of RA?

A

A wide variety of extra-articular complications occur in patients with rheumatoid arthritis (RA):
respiratory: pulmonary fibrosis, pleural effusion, pulmonary nodules, bronchiolitis obliterans, methotrexate pneumonitis, pleurisy
ocular: keratoconjunctivitis sicca (most common), episcleritis, scleritis, corneal ulceration, keratitis, steroid-induced cataracts, chloroquine retinopathy
osteoporosis
ischaemic heart disease: RA carries a similar risk to type 2 diabetes mellitus
increased risk of infections
depression

Less common
Felty’s syndrome (RA + splenomegaly + low white cell count)
amyloidosis

115
Q

What is denosumab?

A

Denosumab is a relatively new treatment for osteoporosis. It is a human monoclonal antibody that prevents the development of osteoclasts by inhibiting RANKL. Remember that osteoblasts build bone, osteoclasts eat bone. It is given as a subcutaneous injection, at a dose of 60mg, every 6 months.

A larger dose of denosumab (120mg) may also be given every 4 weeks for the prevention of skeletal-related events (i.e. pathological fractures) in adults with bone metastases from solid tumours. For example, you may have noticed some of your breast cancer patients have been prescribed denosumab.

Where does it fit in the management of osteoporosis?

Oral bisphosphonates are still given first-line, with oral alendronate being the first-line treatment. If alendronate is not tolerated then NICE recommend using an alternative bisphosphonate - either risedronate or etidronate. Following this the advice becomes more complicated with the next-line medications only being started if certain T score and other risk factor criteria being met. Raloxifene and strontium ranelate were recommended as next-line drugs in the NICE criteria but following recent safety concerns regarding strontium ranelate it is likely there will be an increasing role for denosumab.

NICE published a technology appraisal looking at the role of denosumab in 2010. A link is provided.

What are the known side-effects of denosumab?

Denosumab is generally well tolerated. Dyspnoea and diarrhoea are generally considered the two most common side effects, occuring in around 1 in 10 patients. Other less common side effects include hypocalcaemia and upper respiratory tract infections.

What does the Drug Safety Update add?

Cases of atypical femoral fractures have been noted in patients taking denosumab. Doctors are advised to look out for patients complaining of unusual thigh, hip or groin pain.

116
Q

How does psoriatic arthritis present?

A

Psoriatic arthropathy correlates poorly with cutaneous psoriasis and often precedes the development of skin lesions. Around 10-20% percent of patients with skin lesions develop an arthropathy with males and females being equally affected

Types*
rheumatoid-like polyarthritis: (30-40%, most common type)
asymmetrical oligoarthritis: typically affects hands and feet (20-30%)
sacroilitis
DIP joint disease (10%)
arthritis mutilans (severe deformity fingers/hand, ‘telescoping fingers’)

Management
should be managed by a rheumatologist
treat as rheumatoid arthritis but better prognosis

117
Q

What is Ehler danlo’s syndrome?

A

Ehler-Danlos syndrome is an autosomal dominant connective tissue disorder that mostly affects type III collagen. This results in the tissue being more elastic than normal leading to joint hypermobility and increased elasticity of the skin.

Features and complications
elastic, fragile skin
joint hypermobility: recurrent joint dislocation
easy bruising
aortic regurgitation, mitral valve prolapse and aortic dissection
subarachnoid haemorrhage
angioid retinal streaks

Beighton score is a useful tool to assess hypermobility. Beighton score is positive if at least 5/9 in adults, or at least 6/9 in children. See https://www.ehlers-danlos.com/assessing-joint-hypermobility/ on how to carry out the Beighton score assessment

118
Q

What are the risk factors for pseudo gout?

A

Pseudogout (or “false gout”) is a form of arthritis that results from deposits of calcium pyrophosphate crystals.

Risk factors

Advanced age

Injury or previous joint surgery

Hyperparathyroidism

Haemochromatosis

Hypomagnesaemia

Hypophosphataemia

Presentation

Inflammation and pain in one joint (usually monoarticular arthritis but may be oligoarticular)- commonly the wrist or shoulder that worsens over weeks

Affected joints are acutely inflamed with swelling, effusion, warmth and tenderness

Microscopy Findings

Positively berefringent romboid shaped crystals made of calcium pyrophosphate
Treatment

Acute episodes of pseudogout are treated very similarly to those of gout.

Treatment is usually with a course of NSAIDs.
If NSAIDs are contraindicated, a course of colchicine may be used instead. Colchicine is problematic because it is very prone to causing significant GI disturbances, especially diarrhoea.
If both NSAIDs and colchicine are contraindicated, a short course of oral steroids, or an intra-articular steroid injection may be used.
Unlike gout, there is no effective prophylactic treatment for pseudogout.

119
Q

What is felty’s syndrome?

A

This is the correct answer. Felty’s syndrome is a triad of rheumatoid arthritis, splenomegaly, and neutropenia. Her splenomegaly explains her abdominal discomfort, and her productive cough and coarse crackles suggest a pneumonia, which could be brought on by her neutropenia. Note that splenomegaly can occur in rheumatoid arthritis without Felty’s syndrome, and DMARDs alone can increase susceptibility to infection.

120
Q

Which one of the following is the most appropriate dose of Prednisolone for the patient to be initiated on with GCA?

A

60mg Prednisolone once daily

121
Q

What does skin biopsy of polyarteritis nodosa show?

A

This is the correct answer. A young patient with a history of active hepatitis B presents with systemic upset, nodular and ulcerating skin lesions and mononeuritis multiplex. Skin biopsy will reveal a non-granulomatous necrotising vasculitis.

122
Q

What can cause a rise in both p and c Inca

A

Dual positivity (to both p- and c-ANCA) should raise suspicion of a false positive result. Cause include: subacute bacterial endocarditis (and other chronic infections), haematological malignancy, inflammatory bowel disease, and drug induced vasculitis (including cocaine induced vasculitis).

123
Q

How is a renal crisis treated in systemic sclerosis?

A

Systemic sclerosis can present as limited systemic sclerosis (including CREST) or diffuse systemic sclerosis. The difference is in the skin involvement (limited systemic sclerosis involves skin up to the elbows and knees). The antibody present in the limited form is anti-centromere. The antibodies present in the diffuse form are scl70 and RNA polymerase 1/2/3.

An acute hypertensive crisis is one of the commonest causes of death and is an emergency. These patients are often on a renal ward and may require dialysis due to their renal failure. The treatment is intensive BP control. ACE inhibitors are first line.

124
Q

What drugs can precipitate gout

A

Acute gout is an acute crystal arthropathy, characterised by acute onset joint pain, redness and swelling.

It is more common in men (4-5:1) and in older age (peak incidence at 80 years).

50% of patients present with podagra, a term referring to acute monoarthritis of the first metatarsophalangeal joint. Others commonly involved include the ankle and tarsal joint, though any can be involved.

Predisposing factors can be hereditary or acquired. A positive family history of gout increases risk, as do some uncommon tubular disorders (including Lesch Nyan syndrome).

Acquired risk factors can be considered in terms of increased urate input (diet, cell turnover and metabolic changes) or reduced urate output (chronic kidney disease, endocrine disorders, and medications).

Drugs are a common risk factor for developing gout. The CANT LEAP mnemonic (ciclosporin, alcohol, nicotinic acid, thiazides, loops, ethambutol, aspirin, and pyrazinamide) is a useful aid to help remember some of the common culprits.

125
Q

What drugs can cause lupus and antibodies are elevated?

A

Common culprits include: hydralazine, isoniazid, penicillamine, procainamide, and minocycline.

It is associated with a wide range of serological abnormalities including antinuclear antibodies (ANA), anti-neutrophil cytoplastic antibodies (ANCA), and classically anti-histone antibodies. Treatment is withdrawal of the offending agent. Patients commonly relapse on re-challenging with the causative drug.

126
Q

How can polyarteritis nodosa present in young men?

A

Polyarteritis nodosa is a systemic necrotising vasculitis of small and medium size vessels. It is seen predominantly in young adults with a slight male predominance (2:1) and is associated with chronic viral hepatitis (in particular hepatitis B infection).

It is characterised by prominent non-specific systemic upset with fever, weight loss, arthralgia/myalgia and fatigue. Particularly suggestive symptoms include cutaneous nodules (hence the name) and ulcers, and testicular pain. As with all vasculitis, a mononeuritis multiplex may be seen, as well as renal and GI microvascular ischaemia presenting as renal impairment and gut angina respectively. Unlike small vessel ANCA-associated vasculitis, pulmonary involvement is rare.

On laboratory testing the inflammatory markers are raised, though no single test is particularly suggestive. Positive p-ANCA is seen in 20% of cases. Biopsy of lesions reveals a necrotising vasculitis without granulomas (in contrast to granulomatosis with polyangiitis, and giant cell arteritis which may be on the differential).

Initial treatment of severe disease is with steroids and cyclophosphamide (rituximab may also be used). Diagnosis and treatment of underlying hepatitis B should also be considered.

127
Q

What Investigations are done for polymyositis?What findings are seen and what cancers are associated?How is it treated?

A

Investigations for polymyositis and dermatomyositis

The three most important initial tests to perform on all patients in whom you suspect a diagnosis of dermatomyositis or polymyositis are:

Muscle-derived serum elevation. Creatinine kinase is the most sensitive indicator and the level usually parallels disease activity, particularly in polymyositis.
EMG is performed next. It confirms the myopathic pattern of the disorder and excludes a primary neurogenic disorder. Typical neurogenic records include short duration and low amplitude polyphasic motor units on voluntary activation.
Muscle biopsy is required next for definitive diagnosis.
In polymyositis, muscle biopsy shows endomysial inflammatory infiltrates and in dermatomyositis, it shows perivascular, perimysial inflammatory infiltrate.

Elevated serum enzymes in polymyositis and dermatomyositis

Blood tests that are elevated include lactate dehydrogenase, aldolase, ALT and AST.

The following is a mnemonic to help remember the elevated serum enzymes in both dermatomyositis (DM) and polymyositis (PM):

DM and PM turn your muscles into CLAAA (clay)

Creatine kinase
Lactate dehydrogenase
Aldolase
ALT
AST
Other manifestations of dermatomyositis

Dermatomyositis can manifest as a para-neoplastic syndrome associated with visceral carcinomas - particularly gastric, colorectal, genitourinary and lung cancers.

Therefore, in both polymyositis and dermatomyositis it may be necessary to look for an underlying malignancy with further investigations.

Other manifestations of polymyositis

Polymyositis can also manifest as a para-neoplastic syndrome - particularly in association with lung cancer.

Treatment of polymyositis and dermatomyositis

Corticosteroids are the mainstay of treatment. They are started at high doses initially, creatanine kinase is then monitored to guide the rate of tapering the dose.
Many patients need an additional immunosuppressant such as methotrexate or azathioprine as a steroid-sparing agent.
Hydroxychloroquine occasionally helps with skin disease in dermatomyositis.
As well as pharmacological therapy, physiotherapy is essential to rehabilitate patients.
References

128
Q

What drugs can cause a combined immunodeficiency picture?

A

Lamotrigine

129
Q

What drugs can cause a hypogammaglobinemia?

A

phenytoin

130
Q

What condition can cause absent tonsils?

A

X-linked gammaglobinemia

131
Q

What autoimmune conditions are common in type 2,3 and 4?

A

2-Myasthenia gravis,graves
3-RA,SLE
4-Steven Johnsons, chronic tranplant rejection

132
Q

What is a late phase reaction in allergy?

A
  • T helper 2 cells, and the newly recruited eosinophils and basophils secrete pro-inflammatory mediators, cytokines and chemokines,leucotrienes in the late phase which can trigger a second reaction.(6 to 12 hours of after initial symptoms)
  • Repeat exposure can lead to ongoing local inflammation seen in allergic rhinitis and allergic asthma(structural damage)
133
Q

What are the risk factors for developing anaphylaxis?

A

Background history of asthma or allergy

134
Q

How can anaphylaxis be confirmed

A

triptase-released from activated mast cells(1-2 hours to 4-6 hours after)
check baseline after 24 to exclude mastocytosis

135
Q

How can Anaphylaxis be treated and how is this different to cardiac arrest

A

ABCDE approach

500 micrograms of IM adrenaline 1 in 2000 for anaphylaxis
Other -establish airway, high flow o2,IV fluids

Consider IV 10 mg chlorphenamine and 200mg hydrocortisone

In cardiac arres 1 mg of 1 to 10000 adrenaline is given IV

136
Q

What does thumb-printing indicate?

A

Thumbprinting

Sign of a swollen bowel wall – usually due to inflammation in IBD

137
Q

Why is TNF alpha used for treatment?

A

IFN-alpha can be used to treat chronic hepatitis C infections, Kaposiís sarcoma and hairy cell leukaemia. Interferons are important in certain treatments for viral infections - IFN-alpha can be obtained from multiple sources and limits viral replication.

Note that there are other treatments that are designed to target cytokines, or utilise cytokines as treatment:

Rheumatoid arthritis: Anti-TNF
Renal cell carcinoma: Anti-IL12
Melanoma: Anti-IL12
Neutropenia: GM-CSF or G-CSF

138
Q

How is SCID inherited?

A

Severe Combined Immunodeficiency (SCID) is one of the few primary immunodeficiencies that does not always follow an X-linked inheritance pattern. SCID can follow an X-linked inheritance, most commonly with a mutation in the X-linked common ? Chain which is used by several cytokine receptors. However, SCID can also result from mutations in autosomal genes, such as those encoding the recombinase genes RAG-1 and RAG-2 (which are required to produce immunoglobulin).

SCID is a serious immunodeficiency that presents with both compromised T cell and humoral immunity. Therefore, recurrent infections are likely to lead to investigation and diagnosis.

139
Q

What are the causes of non allergic rhinitis?

A

Important to consider non-allergic causes as even with positive allergy tests if there are other causes present then treatment of allergies may be ineffective in controlling symptoms. Consider:

Infection

Usually self -resolving
Environmental triggers

Smoke, perfume, paint fumes, changes in weather / temperature, alcohol, spicy food, stress and cause not known.
Medicines and recreational drugs

ACE inhibitors, beta blockers, NSAIDs, cocaine
Overuse of nasal decongestants

Reduce swelling of the blood vessels
If used for >5-7 days then rebound symptoms
Hormone changes

Pregnancy, puberty, HRT, OCP
Hypothyroidism
Structural nasal problems

Deviated nasal septum; polpys

140
Q

When is desensitisation immunotherapy done?

A

He had asked his GP about desensitisation therapy

This is only indicated when triple therapy, as outlined above has failed and oral steroids are required.
It is helpful in severe cases and reduces symptoms and need for other medication; some medication may still be required despite immunotherapy
It requires considerable commitment (teatment will be for a minimum of three years, it can be by continuous sublingual therapy or winter courses of 4-6 injections, and treatment effect may wear off and treatment may need to be repeated).

141
Q

When is CMV and irradiated blood products used?

A

Cytomegalovirus (CMV) is transmitted in leucocytes. As most blood products (except granulocyte transfusions) are now leucocyte depleted CMV negative products are rarely required.

Irradiated blood products are depleted of T-lymphocytes and used to avoid transfusion-associated graft versus host disease (TA-GVHD) caused by engraftment of viable donor T lymphocytes.

The table below shows the indications for CMV and irradiated blood:

Situation CMV negative Irradiated
Granulocyte transfusions ✓ ✓
Intra-uterine transfusions ✓ ✓
Neonates up to 28 days post expected date of delivery ✓ ✓
Pregnancy: Elective transfusions during pregnancy (not during labour or delivery) ✓
Bone marrow / stem cell transplants ✓
Immunocompromised (e.g. chemotherapy or congenital) ✓
Patients with/previous Hodgkins Disease ✓
HIV

142
Q

What are the side effects of common cytotoxic agents?

A

Cytotoxic agents

The tables below summarises the mechanism of action and major adverse effects of commonly used cytotoxic agents.

Alkylating agents

Cytotoxic Mechanism of action Adverse effects
Cyclophosphamide Alkylating agent - causes cross-linking in DNA Haemorrhagic cystitis, myelosuppression, transitional cell carcinoma

Cytotoxic antibiotics

Cytotoxic Mechanism of action Adverse effects
Bleomycin Degrades preformed DNA Lung fibrosis
Anthracyclines (e.g doxorubicin) Stabilizes DNA-topoisomerase II complex inhibits DNA & RNA synthesis Cardiomyopathy

Antimetabolites

Cytotoxic Mechanism of action Adverse effects
Methotrexate Inhibits dihydrofolate reductase and thymidylate synthesis Myelosuppression, mucositis, liver fibrosis, lung fibrosis
Fluorouracil (5-FU) Pyrimidine analogue inducing cell cycle arrest and apoptosis by blocking thymidylate synthase (works during S phase) Myelosuppression, mucositis, dermatitis
6-mercaptopurine Purine analogue that is activated by HGPRTase, decreasing purine synthesis Myelosuppression
Cytarabine Pyrimidine antagonist. Interferes with DNA synthesis specifically at the S-phase of the cell cycle and inhibits DNA polymerase Myelosuppression, ataxia

Acts on microtubules

Cytotoxic Mechanism of action Adverse effects
Vincristine, vinblastine Inhibits formation of microtubules Vincristine: Peripheral neuropathy (reversible) , paralytic ileus
Vinblastine: myelosuppression
Docetaxel Prevents microtubule depolymerisation & disassembly, decreasing free tubulin Neutropaenia

Topoisomerase Inhibitors

Cytotoxic Mechanism of action Adverse effects
Irinotecan Inhibits topoisomerase I which prevents relaxation of supercoiled DNA Myelosuppression

Other cytotoxic drugs

Cytotoxic Mechanism of action Adverse effects
Cisplatin Causes cross-linking in DNA Ototoxicity, peripheral neuropathy, hypomagnesaemia
Hydroxyurea (hydroxycarbamide) Inhibits ribonucleotide reductase, decreasing DNA synthesis Myelosuppression

143
Q

What are the important tumour markers for common cancers?

A

Next question

Tumour markers

Tumour markers may be divided into:
monoclonal antibodies against carbohydrate or glycoprotein tumour antigens
tumour antigens
enzymes (alkaline phosphatase, neurone specific enolase)
hormones (e.g. calcitonin, ADH)

It should be noted that tumour markers usually have a low specificity

Monoclonal antibodies

Tumour marker Association
CA 125 Ovarian cancer
CA 19-9 Pancreatic cancer
CA 15-3 Breast cancer

Tumour antigens

Tumour marker Association
Prostate specific antigen (PSA) Prostatic carcinoma
Alpha-feto protein (AFP) Hepatocellular carcinoma, teratoma
Carcinoembryonic antigen (CEA) Colorectal cancer
S-100 Melanoma, schwannomas
Bombesin Small cell lung carcinoma, gastric cancer, neuroblastoma

144
Q

What is the commonest lung cancer type in smokers?

A

Although most lung cancer is linked to smoking, squamous cell lung cancer is the most strongly linked with smoking. This type of lung cancer is also more commonly found near large airways, unlike adenocarcinoma which is more commonly peripheral. Small cell lung cancers also tend to be found near the larger airways. They tend to be associated with para-neoplastic syndromes and often metastasise early. Large cell carcinoma is considered a diagnosis of exclusion. The history is not typical of carcinoid. Furthermore, carcinoid tumours of the lung are rare.

145
Q

What are the different types of non small cell lung cancers?

A

There are three main subtypes of non-small cell lung cancer:

Squamous cell cancer
typically central
associated with parathyroid hormone-related protein (PTHrP) secretion → hypercalcaemia
strongly associated with finger clubbing
hypertrophic pulmonary osteoarthropathy (HPOA)

Adenocarcinoma
typically peripheral
most common type of lung cancer in non-smokers, although the majority of patients who develop lung adenocarcinoma are smokers

Large cell lung carcinoma
typically peripheral
anaplastic, poorly differentiated tumours with a poor prognosis
may secrete β-hCG

146
Q

What are the risk factors for cervical cancer?

A

Human papillomavirus (HPV) subtypes 16,18, 33 are the most significant risk factors for developing cervical cancer. Though it should be noted that the majority of healthy women will spontaneously clear HPV within 2 years without developing cervical cancer.

Smoking is a risk factor for cervical cancer likely through inhibition of the immune system in clearing HPV infection.

Combined oral contraceptives (COC) are implicated in an increase in cervical cancer (and breast cancer), but in the case of cervical cancer risk is increased when taking COC for more than 5 years.

147
Q

What cancer causes raised calcitonin?

A

Medullary thyroid cancer will cause a rise in calcitonin levels. Medullary thyroid cancer is the only thyroid cancer that will cause a rise in calcitonin, as it originates from the parafollicular cells, which produce calcitonin.

148
Q
  • What are the supportive features of Vasovagal syncope?

How is it managed?

A

Supportive features:

  • Triggers:hot and crowded environment, skipped breakfast
  • Pre-syncopal symptoms:nausea and malaise, hot and clammy, looked pale
  • During:brief myoclonic jerks
  • After:fast recovery (10-15min), recognised friends, no prolonged post-ictal phase

It would be worth getting a full set of observations, ECG and systemically examining this lady, but you would expect these to be normal. Given the clear history of vasovagal syncope with triggers, blood tests would not be necessary.

You reassure this lady, give conservative advice about syncope triggers and send her home.

149
Q

What are the supportive features of epileptic seizures

How would this be managed?

A

Supportive features:

  • Before:No warning or pre-syncopal symptoms
  • During:Lasted 1-2 minutes. Load groan noise represents tonic contraction of diaphragm, followed by tonic rigid fall. Description of symmetrical jerking clonic-type movements. Non respsonsive.
  • After:Description of post-ictal period: drowsy and confused, not recognising people around her. 1st clear memory is >1hour later, after already in hospital.

This patient is presenting with her first seizure. You would take a full history including asking about drugs and alcohol, any childhood seizures and any family history. You would do a full neurological examination to look for any focal neurology. You would also need to take a full set of bloods to look for any cause. This would include electrolytes, glucose and lactate. You would need to ask for a 12 -lead ECG.

New focal neurological deficit, head trauma, fevers and persistent headaches would all be reasons to request a CT head now but otherwise you could allow the neurology team to decide this at a later date.

If all history, examination and investigations were normal, it would appear that this lady has had an unprovoked seizure and you would need to inform her that she can no longer drive and will need to discuss the timings of this with a neurologist at her first fit clinic (which you would refer her to). It would be worth checking with your senior if they thought this patient was suitable for discharge.

150
Q

What are the supportive features of cardiac syncope?

How would this be managed?

A

Supportive features:

  • Before:Onsetduringexertion. Sudden onset, with little warning.
  • During:Brief loss of consciousness, in keeping with syncope.
  • After:Rapid recovery after collapse, in keeping with syncope

A thorough history would be key in this patient, including any family history of sudden cardiac death. Examination would look for any murmurs and signs of heart failure. A 12-lead ECG would obviously be essential. Lying and standing blood pressure would also be useful. A full set of bloods would be required to rule out causes such as electrolyte imbalances.

NICE guidelines state (Transient loss of consciousness (‘blackouts’) in over 16s [CG109]):

Refer urgently for cardiovascular assessment, with the referral reviewed and prioritised by an appropriate specialist within 24hours, anyone with TLoC who also has any of the following.

  • An ECG abnormality (see recommendations 1.1.2.2 and 1.1.2.3).
  • Heart failure (history or physical signs).
  • TLoC during exertion.
  • Family history of sudden cardiac death in people aged younger than 40years and/or an inherited cardiac condition.
  • New or unexplained breathlessness.
  • A heart murmur

Consider referring within 24 hours for cardiovascular assessment, as above, anyone aged older than 65years who has experienced TLoC without prodromal symptoms.

This lady would qualify for urgent referral based on transient loss of consciousness during exertion..

151
Q
  • What are the supportive features of dissociative seizure?

How would this be managed?

A

Although this lady would need to be assessed by a neurologist, she does have features suggestive of a dissociative seizure / non-epileptic attack.

Supportive features:

  • Triggers:busy and crowded environment - may be stressful
  • Before:felt short of breath and racing heart - ?adrenaline/stress feeling. Dissociative seizures can be, but are not always triggered by stress/panic.
  • During:retained consciousness - able to hear people around her. Consciousness cannotbe retained during a generalised seizure (but can be during a focal seizure). The feeling of ‘dissociation’ or ‘watching from the outside’ is a common sensation. Often has limb movements which can be hard for witnesses to differentiate from a seizure.
  • After:Able to recognise people around immediately. Feeling groggy or run-down afterwards, 2 days is generally too long for post-ictal.

It is important to note that sufferers of dissociative attacks do not have control over these events and they are often extremely distressing for them.

These are sometimes called non-epileptic attack disorder

Management of this patient is very similar to the management of your patient with a likely epileptic seizure as this is the other differential. You would again take a full history including asking about drugs and alcohol, any childhood seizures and any family history. You would do a full neurological examination, take a set of bloods and request a 12 lead ECG.

They are unlikely to require an urgent CT head but you should check for those red flags. Your working diagnosis may be incorrect.

This patient also requires a referral to the first fit clinic. She needs to be informed that she cannot drive. We have not yet reached a firm diagnosis but even if these do turn out to be dissociative seizures, they are themselves a reason not to drive and she must inform the DVLA.

152
Q

What is chronic pain and what are the different types?

A

Chronic pain is pain that has lastedlonger than 3 months,persisting beyond the expected healing time for an episode of tissue damage. This type of pain can become counter-productive, leading to loss of function and having a significant impact on the health, social and pyschological well-being of a patient.

Chronic pain can have several causes:

  • Nociceptive pain:Noci (pain) -ceptive (perception). Usually due to an organic disease. Pain that is triggered by stimuli that damage or threaten tissues. The most common of these is musculoskeletal e.g. osteoarthritis. This also includes cancer pain, visceral pain etc.
  • Neuropathic pain: Neuro (nerves) -pathy (disease). Due to pathology or pathological response of sensory nervous system e.g. peripheral neuropathy, MS, post-herpetic pain etc.
  • Nociplastic pain:Noci (pain)-plastic (change/modulation).**Caused by abnormal pain processing, even when no physical/organic damage has occurred to our tissues or nerves. This term was introduced by the IASP council in 2017.

“pain that arises from altered nociception despite no clear evidence of actual or threatened tissue damage causing the activation of peripheral nociceptors or evidence for disease or lesion of the somatosensory system causing the pain.”

A patient can experience more than one type of pain at once. In fact, chronic nociceptive or neuropathic pain can lead to neuromodulation and increased nociplastic pain.

153
Q

What are the risk factors for chronic pain?

A

Chronic pain is more prevalent in some demographic groups, due to a combination of biological, psychological and social factors:

  • Older age groups
  • Co-morbid disease: particularly depression and cardiovascular disease
  • Female gender
  • Socio-economic deprivation
  • Smokers
  • Alcohol dependence
  • Increased BMI
154
Q

How does pain occur?

A
  • Central Sensitisation:increased responsiveness of nociceptors in the central nervous system. It has been proposed that previous painful experiences increase this sensitisation process. This can lead to usually non-painful stimuli (e.g. gut peristalsis in IBS) being processed as painful.
  • Reduced negative feedback:Reduced negative inhibition via the descending pain pathways. Because of the close ties with the cortex and limbic system, it is suggested that states of depression/anxiety/stress inhibit this system, leading to reduced pain inhibition.
  • Somatisation:Slightly controversial. Manifestation of psychological distress as physical pain or symptoms (similar to stress being felt as a headache). Becomes pathological when prolonged and distressing.

Ascending and descending pathways

which drugs act?

155
Q

What are maladaptive health behaviours and yellow flags?

A

Yellow flagsare our risk factors for developing chronic pain:

  • Maladaptive health behaviours
  • Adopting ‘passive’ rather than ‘active’ coping strategies
  • Anxiety
  • Depression and social isolation
  • Lack of social support or ‘over-protective’ caregivers
  • Ongoing litigation related to original pain/injury source
  • Unresolved anger
  • Poor job satisfaction or extended sick leave

Each person’s reaction to pain is different. A person’s health behaviours often contribute to their ability to cope with and recover from chronic pain.

Example ofmaladaptive health beliefs and behavioursinclude:

  • The belief that feeling pain means harm is occurring
  • Fear-avoidance of activities due to fear of pain/re-injury
  • Catastrophic thinking
  • Reliance on passive coping strategies (e.g. drugs and rest) rather than active (e.g. physiotherpay)
  • Low expectations of recovery
  • Adopting “the sick role”
156
Q
  • What is chronic fatigue syndrome?
A
  • What is chronic fatigue syndrome?Chronic Fatigue Syndrome (sometimes called Myalgic Encephalomyelitis/ME) causes long-standing fatigue. It is not always associated with chronic pain but raises similar symptom clusters and diagnostic challenges to conditions such as fibromyalgia
    . CFS is when the level of fatigue is disabling in it’s frequency and severity. There are several different ways to classify chronic fatigue. NICE defines the fatigue as:
    • Present for >4 months
    • New or clear onset
    • Not explained by anything else (including severe obesity BMI >40)
    • Causes significant reduction in activity level
    • Classically post-exertional (typically delayed onset, and lasts >24 hours)
    The patient may be very fatigued by what can seem like ‘trivial’ physical/mental exertion, and would often have been within the patients activity tolerance prior to onset of the disease.CFS is also associated withat least one of:
    • Difficulty sleeping
    • Widespread muscle/joint pain
    • Cognitive impairment i.e. poor memory, word-finding difficulties, poor concentration
    • Ongoing flu-like symptoms: malaise, painful lymph nodes, sore throat, headaches
    • Dizziness/nausea/palpitations/orthostatic intolerance
    Additionally,the symptoms are not explained by an alternative diagnosis e.g. depression, arthritis, malignancy etc.Remember, fatigue is not the same as depression. You may have to clarify the symptoms during the history to elicit the patient’s meaning.
    • *Fatigue:**Reduced energy, subjective feeling of tiredness or exhaustion.
    • *Depression:**Reduced motivation and enjoyment. May feel down/weepy/upset. May be associated with concomitant fatigue.
    CFS can be present at the same time as commonly associated conditions such as IBS, migraine and fibromyalgia.
157
Q

What Investigations are done for chronic fatigue syndrome?

A

CFS very often has a ‘triggering event’ such as infection with EBV, other viral infections, vaccination commonly at a time of stress/exertion. It is not known what causes CFS and may theories have been investigated. There has been suggested neuroinflammatory and neuroendocrine involvement, and research into this is ongoing.

Up until this time, there is no diagnostic blood tests, antibody or imaging modality that can diagnose CFS so it remains based on clinical acumen and ruling out differential diagnoses.

A patient presenting with fatigue should undergo routine ‘Tired all the time’ (TATT) work-up plus any additional investigations indicated by the specific features of the history. Suggested tests might include:

  • FBC
  • U&Es
  • LFTs
  • TFTs
  • CRP/ESR
  • HbA1c
  • IgA anti-tissue transglutaminse (anti-ttg) - coeliac disease can cause fatigue without GI symptoms

Consider if suggestive history:

  • Bone panel/myeloma screen
  • Vitamin D
  • EBV/CMV
  • Lyme serology
  • HIV/Hepatitis
158
Q

How to manage cfs?

A

NICE has previously recommended CBT and graded exercise therapy for CFS, but this remains a very controversial topic. Therapies for CFS are considered ascoping therapiesnot curative therapies.

The basis of treatment often involves supporting a patient to understand the disease and to find adaptations to cope with it and maximise their quality of life and functioning.

Patient often report their symptoms vary day to day. Some patients report a tendency to “over-exert” themselves when experiencing a “good day” leading to days-weeks of increased fatigue. Patients are taughtpacingtechniques as a coping strategy to avoid this ‘boom-bust’ cycle. The principles of pacing are avoiding over-exertion, tracking activity and symptoms and varying activities.

159
Q

What is complex regional pain syndromes?

A

Complex regional pain syndrome (CRPS) is rarely encountered in clinical practice, but is good to at least be aware of.

The syndrome typically affects a distal limb, after an injury such as a fracture, surgery or trauma. The area affected goes on to develop chronic severe and debilitating pain, in excess of what would be expected from the original injury. Symptoms usually start within 1 month of the injury. It is thought to originate fro damage to either 1 major nerve or to the small sensory and autonomic nerves in an area.

Unlike fibromyalgia and CFS there are often visible clinical signs at the affected limb, due to damage to the autonomic control of vasodilation, sweating and temperature.

Sensory-allodynia,hyperalgesia,
temperature and colour changes
edema,reduced motor function

Diagnosis is clinical. Nerve conduction studies and MRI can sometimes show nerve damage, but a normal scan dose not rule out CRPS.

The pain is often so severe that the limb is rendered non-functional. It is not uncommon for patients to request amputation of the affected limb. Unfortunately, this has not been found to be curative of the pain and often leads to the development of phantom limb syndrome.

160
Q

What features may lead to initial resistance to diagnosis

A
  • Lack of understanding on behalf of patient
  • Unclear or unempathetic explanation from clinician
  • Felt stigma surrounding a diagnosis - feeling of stigma from clinicians/society
  • Feeling of being accused of “making it up” or “saying it’s all my head”
  • Feeling that symptoms are “just being put down to being depressed”
  • Feeling that pain/symptoms are not being validated as real/significant
  • Feeling that another diagnosis is being missed
  • Lack of cure/treatment

Alternatively, diagnosis can be really positive experience for some patients. Many fibromyalgia patients have waited for over 5 years before receiving their diagnosis. Receiving a diagnosis can offer a patients an explanation for their symptoms after years of uncertainty and distress. Having a diagnosis can allow patients to explain their disease to family/friends and join support groups.

161
Q

How can chronic pain be managed?

A
  • Felt stigma surrounding a diagnosis - feeling of stigma from clinicians/society

t is important to identify features in chronic pain which predispose, precipitate and perpetuate chronic pain. For example, a pain condition may cause pain at night and lead to poor sleep. Un-refreshing sleep and chronic tiredness, in turn, leads to increased pain perception. A part of management is to identity these self-perpetuating cycles and offer the patient support to break them, through a mixture medication, education, physiotherapy and CBT.

162
Q

How does tens reduce pain?

A

Summary: mechanicalstimulation inhibits pain signalling

TENs (transcutaneous electrical nerve stimulation) machines are used to manage both acute and chronic pain. They work via sticky pads on the skin that give small electrical currents, feeling like a ‘buzz’ or ‘tingling sensation’. This acts to interrupting painful signals, thus the intensity of the pain is reduced.

They are used for acute pain, such as labour pain. There is little evidence to support their use for chronic pain but some patients may choose to try them, particularly for focal pain such as chronic back pain.

163
Q

What is IIness anxiety disorder?

A

Illness anxiety disorder and medically unexplained symptoms can appear superficially similar but are distinct conditions. They can certainly overlap and a patient can experience both simultaneously.

Illness anxiety disorder is usually described as a cycle of recurrent anxiety surrounding fears of ill-health. This can lead to healthcare avoidance or recurrent presentations to health services despite reassurance.

Someone with illness anxiety disorder may be experiencing organic physical symptom, medically unexplained symptoms or no symptoms at all.

Hyper-vigilance of bodily functions and sensory sensitisation can lead a patient with illness anxiety to develop new physical symptoms.

People with medically unexplained symptoms may not have any associated illness anxiety. People experiencing medically unexplained symptoms often are concerned about specific possible diseases but may react well to reassurance and explanation, without demonstrating a recurrent cycle of illness anxiety.

164
Q

What are the harms of over investigation?

A
  • Delay in reaching a diagnosis, patient acceptance and management.
  • Perpetuate patient and clinician belief in an underlying organic disease, making later acceptance of medically unexplained symptoms more challenging.
  • Repeated negative results lead to patient understanding of self as a “medical mystery” who needs prolonged and increasingly specialist investigations or care to unveil the diagnosis.
  • Precipitate or perpetuate illness anxiety, or put patients through stress of awaiting results which may reveal a serious diagnosis e.g. cancer, despite being extremely unlikely.
  • Dis-empower patients and lead to an increased external locus of control, increasing dependence on healthcare and clinicians.
  • Repeated scans can increase radiation exposure.
  • Iatrogenic harm through invasive investigations e.g. biopsies, surgery.
  • Unnecessary costs to the NHS.
165
Q

What can cause begin palpitations and how can functional palpitations be treated?

A

Ectopic beats and sinus tachycardia can be common causes of palpitations that do not affect a patient’s prognosis.

Ectopics- can be atrial or ventricular. Feels like a ‘missed beat’ then a ‘heavy squeeze’.

  • Atrial ectopics are common and does not affect prognosis.
  • Ventricular ectopics can be associated with increased caffeine intake or stress - reducing caffeine and stress often improves palpitations in these patients. Ventricular ectopics also tend to be more common with increased age. A heavy burden of ventricular ectopics (e.g. bigeminy) can indicative of electrolyte disturbances, underlying cardiac ischaemia or structural abnormalities and warrants further investigation.

Sinus tachycardia- a racing heart beat, but the rhythm is regular. Can be a physiological response to underlying pathology such as sepsis or PE. Sinus tachycardia is also associated with anxiety as the increase in catecholamine levels lead to increased heart rates.

When serious disease has been ruled out, NICE recommends treating palpitations with reassurance, lifestyle advice and managing any cardiac risk factors e.g. hypertension, hypercholesterolaemia.

Palpitations are a scary experience for patients, sometimes a single episode of palpitations can lead to anxiety and fear - “My heart is giving up!”. This, in turn, leads to increased vigilance of their heart beat and anxiety that leads to further tachycardia.

Exploring anxiety and life stressors sensitively can help a patient to identify features that could be causing/contributing to their symptoms. Relaxation, mindfulness and coping strategies can be helpful in reducing palpitation frequency and severity.

166
Q

What are the features of IBS

A

Irritable bowel syndrome

Irritable bowel syndrome (IBS) is one of the most well-recognised cause of functional abdominal pain. It is common and well-described, thought to affect 10-20% of the population. It is defined as afunctional bowel conditionas thefunctioningof the bowel has been impaired - causing altered bowel habit and pain. Several theories have been proposed, however, thus far no organic pathology has been identified and examination and investigations are generally normal.

IBS diagnosis should be approached based on a patientpositivelyhaving IBS symptoms, not simply as a diagnosis of exclusion once organic disease has been ruled out.

IBS is characterised by at least 6+ months of either:

  • A:Abdominal pain (cramping) - often relieved by defecation
  • B:Bloating
  • C:Constipation (+/- diarrhoea) - altered bowel frequency or form, mucous may be present in stool

IBS is also associated with lethargy, nausea, backache and bladder dysfunction. 1 in 10 patient report IBS onset can develop after an episode of gastroenteritis. Depression and anxiety are more common amoungst IBS patients.

People with IBS tend to have a predominantly constipation-type, diarrhoea-type or mixed.

167
Q

What is Functional dyspepsia?

A

Functional dyspepsia is an issue with upper GI pain and delayed gastric emptying. Symptoms that are primarilyupper gastrointestinal.Symptoms include:

  • Dyspepsia
  • Nausea
  • Early satiety
  • Bloating
  • Increased burping

These symptoms clearly also raise some red flags for upper GI disease and many patients will be investigated with bloods, H Pylori testing and often endoscopy - looking for ulcers, inflammation, malignancy etc. You should screen for red flags and ALARM symptoms in any patient with dyspepsia.

Functional dyspepsia is diagnosed once organic diseases that could also explain the symptoms have been ruled out. Up to 60% of people with recurrent dyspepsia will have functional dyspepsia, it has previously been called ‘non-ulcer dyspepsia’. 1 in 3 people with functional dyspepsia also have IBS.

168
Q

What is centrally mediates abdominal pain?

What is abdominal migraine?

A

Centrally-mediated (functional) abdominal pain syndrome:

> 6 months of abdominal pain, with no explained organic cause.

The Rome III criteria defines the pain as:

  • Pain is nearly continuous for at least 3 months
  • Not, or minimally, related to eating/defecation/menses
  • Impairs daily functioning
  • Not attributable to another medical condition

Prevalence is thought to be around 2% in adults. More common in females and younger adults/adolescents. Increased prevalence in patients with life/social stressors, which can be a precipitating or perpetuating factor.

Abdominal migraine is a functional abdominal pain condition, primarily found in children.

  • Paroxysmal episodes of central abdominal pain, lasting >1 hour.
  • Associated with: nausea, vomiting, loss of appetite and sometimes headache/photophobia.
  • Long symptom-free periods in between episodes.

Children with abdominal migraines may go to develop headache migraines in adolescence/adulthood.

Abdominal migraine is predominantly noted in in paediatrics, but you may by interested in thisinteresting case reportabout a patient with recurrent abdominal pain and migraines whose pain attacks resolved when placed on migraine prophylaxis.

169
Q

What is chronic pelvic pain?

What are the red flags?

A

This is chronic pain, lasting >6months, in the lower abdomen or pelvic area. The pain is not purely associated with menstruation, intercourse or pregnancy. This is generally a gynecological term, referring to women, or anyone with a uterus/ovaries. These patients may be seen by gynaecology, general surgery and primary care. Male chronic pelvic pain syndrome also exists, with both organic and functional causes, but is not the focus of this tutorial.

Approach to chronic pelvic pain takes on a similar approach to abdominal pain. There are important organic pathologies to consider and investigate for, but for many patients investigations are normal. These cases are considered as functional pelvic pain. Pelvic pain can be associated with abnormal uterine bleeding, urinary symptoms and constipation.

Some organic pelvic conditions e.g. endometriosis can be challenging to diagnose, this can make differentiating functional and organic pelvic pain fairly challenging.

Similarly to functional abdominal pain, the pain may be a result of central sensitisation, biopyschosocial factors and contributing organic pathology.

Red flags for pelvic pain:

  • Post-coital bleeding
  • Post-menopausal bleeding
  • PR bleeding
  • New GI symptoms e.g. constipation, bloating, early satiety, especially age >50
  • Pelvic mass
  • Unexpected weight loss
170
Q

What is endometriosis

A

Pelvic pain with a significant cyclical element (worsening pre/during menstruation) may be due to endometriosis. This is an organic condition caused by endometrial tissue growth at extra-uterine sites, leading to dysmenorrhea, pelvic pain and sometimes subfertility. It can be difficult to differentiate endometriosis from functional pelvic pain due to the investigations required.

Endometriosis poses a diagnostic challenge as the gold-standard for proven diagnosis is to perform a laparoscopy. There an average of 7.5 years between symptom onset and diagnosis. Laparoscopy is an invasive investigation which carries a2.4 in 1000risk of damage to bowel, bladder or blood vessel. There is also a risk of adhesions after an invasive surgery, which could in turn lead to worsening of chronic pelvic pain. The benefits and risks of undergoing laparoscopy to diagnose endometriosis should be explained to patients undergoing investigations.

Laparoscopy is not 100% sensitive and patients should be aware that they may have a normal result, even with endometriosis. Similarly, some women with functional pelvic pain without endometriosis may have similar symptoms, and risk undergoing a potentially harmful and invasive investigation that will not lead to eventual benefit.1/2 to 1/3of diagnostic laparoscopies will be negative.

171
Q

Where can current use of AI be seen?

A

Need for rigorous clinical validation
Ethical implications of ongoing monitoring (wearable devices etc)
The need to educate and prepare augmented doctors
Avoiding dehumanizing by technology – ambient clinical intelligence

172
Q

When should death be reffered to a coroner?

A

It is the statutory duty of a doctor to report a death to the Coroner if the death occurred in that Coroner’s area and is of the sort that needs reporting to the Coroner

Not attended by a doctor during final illness

  1. Not seen by a doctor either after death or within 14 days prior to death
  2. Cause of death unknown
  3. Cause of death may be unnatural
  4. Others
    • in custody
    • war pension
173
Q

What is classified an unatural death?

A
1. Criminal
Murder
Manslaughter
Infanticide
Aiding Suicide
Dangerous or Careless Driving
Suspicious
  1. Suicide
    Accident
    - no matter what the time interval
    RTC’s
    domestic
    industrial
  2. Industrial Disease (pneumoconiosis, asbestosis)
5.  Deaths associated with Medical or Surgical Treatment
During a medical or surgical procedure
operation/anaesthetic
24 hour rule
admission to hospital
Where treatment caused, contributed to or accelerated death
“Negligence”
6.  Miscellaneous
Abortion
Neglect
Acute alcohol
Drug abuse
Children
174
Q

What are the types of autopsies?

A

Medico-legal (Coroner’s; Forensic)
Consented (Hospital)

Can only happen with informed consent from next of kin
Regulated by the Human Tissue Authority and covered by The Human Tissue Act 2004
Next of Kin has to understand
What an autopsy entails
Why an autopsy is being requested
The limitations of the autopsy
When to expect a result
Next of kin must decide
The extent of the autopsy
The extent of tissue retention
What will happen to any retained tissue
175
Q

What is the human tissue act ?

A
“a framework for regulating the storage and use of human organs and tissue from the living and deceased”
Replaces
Human tissue act 1961
Anatomy act 1984
Human organ transplants act 1989

Informed Consent above all
Anything which includes cells
Penalties of up to 3 years in prison

Human Tissue Authority

The human tissue authority
Issue codes of practice
Inform the public
License activities
Inspect premises
176
Q

How is an autopsy done?

A
Clinical History
External examination	confirm identity
Evisceration	Record findings
Organ dissection
Further tests
Report